Giddens, Concept 23, Immunity

Your page rank:

Total word count: 17013
Pages: 62

Calculate the Price

- -
275 words
Looking for Expert Opinion?
Let us have a look at your work and suggest how to improve it!
Get a Consultant

During the neurologic assessment of a client with a tentative diagnosis of Guillain-Barré syndrome, what does the nurse expect the client to manifest?
1
Diminished visual acuity
2
Increased muscular weakness
3
Pronounced muscular atrophy
4
Impairment in cognitive reasoning

2 Muscular weakness with paralysis results from impaired nerve conduction because the motor nerves become demyelinated. Diminished visual acuity usually is not a problem; motor loss is greater than sensory loss, with paresthesia of the extremities being the most frequent sensory loss. Demyelination occurs rapidly early in the disease, and the muscles will not have had time to atrophy; this can occur later if rehabilitation is delayed. Only the peripheral nerves are involved; the central nervous system is unaffected.

The echoviruses can cause which diseases in clients? Select all that apply.
1
Parotitis
2
Gastroenteritis
3
Mononucleosis
4
Aseptic meningitis
5
Burkitt’s lymphoma

2, 4 Echoviruses cause gastroenteritis and aseptic meningitis. Parotitis is caused by mumps. Burkitt’s lymphoma and mononucleosis are caused by the Epstein-Barr virus.

A 5-year-old-child is undergoing chemotherapy. The mother tells the nurse that the child is not up to date on the required immunizations for school. What is the best response by the nurse?
1
"By this time your child has developed sufficient antibodies to provide immunity."
2
"Maintaining current immunizations is critical. Make sure the series is completed."
3
"This isn’t the best time to finish the immunizations, because your child’s immune system is suppressed."
4
"It’s important to complete the immunizations because your child needs to be protected from childhood diseases that could be fatal."

3 Chemotherapy compromises the immune system. The vaccines may be administered after the completion of the chemotherapy protocol, once the immune system has returned to its previous state. The child has not developed sufficient antibodies; booster immunizations are needed, but not at this time. Administering immunizations at this time could prove fatal.

A nurse is reviewing the laboratory reports of four clients. Which client’s laboratory report indicates acquired immunodeficiency syndrome (AIDS)?
1
Client 1, CD4 count, 750 cells/mm
2
Client 2, CD4 count, 550 cells/mm
3
Client 3, CD4 count, 175 cells/mm
4
Client 4, CD4 count, 450 cells/mm

3 The diagnosis of AIDS requires that the person should be HIV positive and have either a CD4+ T-cell count of less than 200 cells/mm 3 (200 cells/uL) or less than 14% or an opportunistic infection. Therefore client 3, with a CD4+ T-cell count of less than 200 cells/mm 3 (200 cells/uL) and who is HIV positive, is having AIDS-defining illness. A healthy client usually has at least 800 to 1000 CD4+ T-cells per cubic millimeter (mm 3) of blood. This number is reduced in the client with HIV disease. Client 1, with a CD4+ T-cell count of 750 cells/mm 3 and HIV positive, does not have AIDS. Client 2, with a CD4+ T-cell count of 550 cells/mm 3 and HIV positive, does not have AIDS. Client 4, having a CD4+ T-cell count of 450 cells/mm 3 and HIV positive does not have AIDS.

The bacteria Clostridium botulinum causes which condition in a client?
1
Upper respiratory tract infection
2
Toxic shock syndrome
3
Urinary tract infection
4
Food poisoning with progressive muscle paralysis

4 Clostridium botulinum bacteria causes food poisoning with progressive muscle paralysis. Toxic shock syndrome is caused by the bacteria Staphylococcus aureus. Many viruses and bacteria can cause upper respiratory tract infection but Clostridium is not one of them. Klebsiella-Enterobacter organisms most likely cause urinary tract infections.

A nurse is teaching a health class about human immunodeficiency virus (HIV). Which basic methods are used to reduce the incidence of HIV transmission? Select all that apply.
1
Using condoms
2
Using separate toilets
3
Practicing sexual abstinence
4
Preventing direct casual contacts
5
Sterilizing the household utensils

1, 3 HIV is found in body fluids such as blood, semen, vaginal secretions, breast milk, amniotic fluid, urine, feces, saliva, tears, and cerebrospinal fluid. Therefore a client should use condoms to prevent contact between the vaginal mucus membranes and semen. Practicing sexual abstinence is the best method to prevent transmission of the virus. The HIV virus is not transmitted by sharing the same toilet facilities, casual contacts such as shaking hands and kissing, or by sharing the same household utensils.

A nurse is caring for a client with Guillain-Barré syndrome. The nurse should prepare the client for what essential care related to rehabilitation?
1
Physical therapy
2
Speech exercises
3
Fitting with a vertebral brace
4
Follow-up on cataract progression

1 Rehabilitation needs for a client with Guillain-Barré syndrome focus on physical therapy and exercise for the lower extremities because of muscle weakness and discomfort. A client with Guillain-Barré syndrome does not need speech or swallowing exercises. A client with Guillain-Barré syndrome does not need vertebral support. Problems with cataracts are not associated with Guillain-Barré syndrome.

A nurse instills an antibiotic ophthalmic ointment into a newborn’s eyes. What condition does this medication prevent?
1
Herpetic ophthalmia
2
Retinopathy of prematurity
3
Ophthalmia neonatorum
4
Hemorrhagic conjunctivitis

3 Ophthalmia neonatorum is caused by gonorrheal and/or chlamydial infections present in the vaginal tract. It is preventable with the prophylactic use of an antibiotic ophthalmic ointment applied to the neonate’s eyes. Herpes affects the neonate systemically. Retinopathy of prematurity (formerly retrolental fibroplasia) occurs as a result of prolonged exposure to a too-high oxygen concentration. Hemorrhagic conjunctivitis is usually caused by rapid expulsion of the fetus’s head from the vagina.

Which drug treats hay fever by preventing leukotriene synthesis?
1
Zileuton
2
Cromolyn sodium
3
Chlorpheniramine
4
Diphenhydramine

1 Zileuton [1] [2] is a leukotriene antagonist drug; this substance prevents the synthesis of leukotrienes and helps in managing and preventing hay fever. Cromolyn sodium stabilizes mast cells and prevents the opening of mast cell membranes in response to allergens binding to immunoglobulin E.. Chlorpheniramine and diphenhydramine are antihistamines and prevent the binding of histamine to receptor cells and decrease allergic manifestations.

What is the role of the plasma cell in the antigen-antibody response?
1
Makes an antigen harmless without destroying it
2
Produces antibodies against the sensitizing antigen
3
Produces antibodies after an exposure to a known antigen
4
Clumps antibody-antigens linkages together to form immune complexes

2 In the antigen-antibody response, once the B-cell is sensitized, it divides and forms a plasma cell, which produces antibodies against the sensitizing antigen. Inactivation or neutralization is the process of making an antigen harmless without destroying it. Memory cells produce antibodies after the next exposure to an antigen that is recognized by the body. Agglutination is the clumping of antigens linked with antibodies, forming immune complexes.

A child who recently returned from a three-day camping trip over spring vacation is brought to the clinic after a rash, chills, and low-grade fever develop. What are the most important data for the nurse to assess when taking the child’s history? Select all that apply.
1
Date of return to school
2
Sports played on camping trip
3
Tendency to allergic reactions
4
Duration of signs and symptoms
5
Recent exposure to poison oak or ivy

3, 4, 5 It is important to know whether the signs and symptoms are related to a history of allergies, a communicable infection contracted during the trip, or some other factor. The nurse must gather information regarding the duration of signs and symptoms because they could be related to a variety of factors that may or may not be linked to the camping trip. It is important to determine whether the child was exposed to a known allergen so appropriate treatment may be initiated. It is not necessary to know when the child is expected back in school; this information is unrelated to the situation. The child’s problem is also unrelated to sports activities.

Which client has the highest risk for human immunodeficiency virus (HIV) infection?
1
A client who is involved in mutual masturbation
2
A client who undergoes voluntary prenatal HIV testing
3
A client who shares equipment to snort or smoke drugs
4
A client who engages in insertive sex with a non-infective partner

3 Clients who use equipment to snort (straws) and smoke (pipes) drugs are at the highest risk for becoming infected with HIV as their judgment may be impaired regarding the high-risk behaviors. Safe activities that prevent the risk of contracting HIV include mutual masturbation, masturbation, and other activities that meet the "no contact" requirements. A client who undergoes perinatal HIV voluntary testing may reduce the chances of getting infected. Insertive sex between partners who are not infected with HIV are not at risk of becoming infected with HIV.

Which vaccine is administered orally in children?
1
MMR vaccine
2
Rotavirus vaccine
3
Live influenza vaccine
4
Meningococcal conjugate vaccine (MCV4)

2 Rotavirus vaccines are generally administered orally because these live viruses should replicate in the gut of the infant. MMR vaccines are generally administered subcutaneously in the upper region of the arm. Live influenza vaccines are administered nasally. Meningococcal conjugate vaccines (MCV4) are administered intramuscularly in the deltoid region.

Which influenza vaccine is administered by way of the intranasal route?
1
Fluarix
2
Fluvirin
3
FluMist
4
Fluzone

3 FluMist is given intranasally. Fluarix, Fluvirin, and Fluzone are also influenza vaccines administered via the intramuscular route.

A client asks the nurse what advantage breast-feeding holds over formula feeding. What major group of substances in human milk are of special importance to the newborn and cannot be reproduced in a bottle formula?
1
Amino acids
2
Gamma globulins
3
Essential electrolytes
4
Complex carbohydrates

2 The antibodies in human milk provide the infant with immunity against all or most of the pathogens that the mother has encountered. Amino acids and essential electrolytes are present in commercial formulas. Complex carbohydrates are not required by the infant.

The nurse is reviewing blood screening tests of the immune system of a client with acquired immunodeficiency syndrome (AIDS). What does the nurse expect to find?
1
A decrease in CD4 T cells
2
An increase in thymic hormones
3
An increase in immunoglobulin E
4
A decrease in the serum level of glucose-6-phosphate dehydrogenase

1 The human immunodeficiency virus (HIV) infects helper T-cell lymphocytes; therefore 300 or fewer CD4 T cells per cubic millimeter of blood or CD4 cells accounting for less than 20% of lymphocytes is suggestive of AIDS. The thymic hormones necessary for T-cell growth are decreased. An increase in immunoglobulin E is associated with allergies and parasitic infections. A decrease in the serum level of glucose-6-phosphate dehydrogenase is associated with drug-induced hemolytic anemia and hemolytic disease of the newborn.

Which vaccine may cause intussusception in children?
1
Rotavirus
2
Hepatitis
3
Measles, mumps, and rubella
4
Diphtheria, tetanus, and pertussis

1 Rotavirus vaccines very rarely cause intussusception, a form of bowel obstruction in which the bowel telescopes in on itself. Hepatitis vaccines can cause anaphylactic reactions. The measles, mumps, and rubella vaccine may cause thrombocytopenia. The diphtheria, tetanus, and pertussis vaccine carries a small risk of causing acute encephalopathy, convulsions, and a shock-like state.

The parents of an infant ask a nurse why their baby is not receiving the measles, mumps, and rubella (MMR) vaccine at the same time that other immunizations are being given. What explanation should the nurse give about why the MMR vaccine is administered at 12 to 15 months of age?
1
There is an increased risk of side effects in infants.
2
Maternal antibodies provide immunity for about 1 year.
3
It interferes with the effectiveness of vaccines given during infancy.
4
There are rare instances of these infections occurring during the first year of life.

2 Maternal antibodies to measles, mumps, and rubella infection persist in the infant until approximately 15 months of age. Side effects are no more common among infants than in toddlers. The measles vaccination does not interfere with the effectiveness of other vaccines. Although the measles, mumps, and rubella do occasionally occur after the administration of the MMR vaccine during the first year of life, the vaccine is not given during this time because of the presence of maternal antibodies.

Why would a client with acquired immunodeficiency syndrome (AIDS) be administered pregabalin?
1
To reduce neuropathic pain
2
To reduce cognitive difficulty
3
To reduce swallowing difficulty
4
To reduce muscle and joint pain

1 Pregabalin is indicated for neuropathic pain based on its mechanism of interference with nerve signaling. Clients with AIDS generally exhibit emotional and behavioral changes, which can be managed with appropriate antidepressants and anxiolytics. AIDS clients who experience difficulty swallowing may have candidal esophagitis; this condition can be managed with antifungal mediations such as fluconazole or amphotericin B. Traditional analgesics are used to manage joint and muscle pain.

Which vaccine is administrated through the intranasal route?
1
Rotavirus vaccine
2
Influenza (live) vaccine
3
Varicella virus vaccine
4
Human papillomavirus vaccine

2 Influenza (live) vaccine is administered through the intranasal route. The rotavirus vaccine is administered orally. The varicella virus vaccine is given as a subcutaneous injection. The human papillomavirus vaccine is given as intramuscular injection.

The parent of a newborn asks a nurse why, except for hepatitis B vaccine, the immunization schedule does not start until the infant is 2 months old. How should the nurse respond?
1
"A newborn’s spleen can’t produce efficient antibodies."
2
"Infants younger than 2 months are rarely exposed to infectious disease."
3
"The immunization will attack the infant’s immature immune system and cause the disease."
4
"Maternal antibodies interfere with the development of active antibodies by the infant when immunized."

4 Passive antibodies received from the mother will be diminished by age 8 weeks and will no longer interfere with the development of active immunity to most communicable diseases. The spleen does not produce antibodies. Infants often are exposed to infectious diseases. The viruses in immunizations are inactivated or attenuated; they may cause irritability and fever but will not cause the related disease.

Which vaccination is given to young children to provide protection against tetanus and diphtheria but not pertussis?
1
Td
2
DT
3
DTaP
4
Tdap

2 DT is given to children to provide protection from both tetanus and diphtheria. Td is used as a booster dose to protect adolescents and adults from tetanus and diphtheria. DTaP is given to children to provide protection from tetanus, diphtheria, and acellular pertussis. Tdap is used as a booster dose to protect adolescents and adults from tetanus, diphtheria, and acellular pertussis.

The nurse is caring for a client who is receiving azathioprine, cyclosporine, and prednisone before receiving a kidney transplant. What does the nurse identify as the purpose of these drugs?
1
Stimulate leukocytosis
2
Provide passive immunity
3
Prevent iatrogenic infection
4
Reduce antibody production

4 These drugs suppress the immune system, decreasing the body’s production of antibodies in response to the new organ, which acts as an antigen. These drugs decrease the risk of rejection. These drugs inhibit leukocytosis. These drugs do not provide immunity; they interfere with natural immune responses. Because these drugs suppress the immune system, they increase the risk of infection.

A teenager with allergies is using oxymetazoline nasal spray. What effect should the nurse assess the client for if more than the recommended dose is taken?
1
Nasal polyps
2
Ringing in the ears
3
Bleeding tendencies
4
Increased nasal congestion

4 With frequent and continued use, oxymetazoline can cause rebound congestion of mucous membranes. Nasal polyps may be associated with allergies but are unrelated to nasal spray use. Ringing in the ears (tinnitus) is not associated with oxymetazoline, although this medication may cause hypotension, tachycardia, and dizziness. Bleeding tendencies are related to inadequate clotting mechanisms, which are not associated with the use of this nasal spray.

Which cells are affected in DiGeorge syndrome?
1
T-cells
2
B-cells
3
Monocytes
4
Polymorphonuclear cells

1 DiGeorge syndrome is a primary immune deficiency disorder in which T-cells are affected. The B-cells are affected in Bruton’s X-linked agammaglobulinemia; common variable hypogammaglobulinemia; and selective IgA, IgM, and IgG deficiency. Monocytes and polymorphonuclear cells are affected in chronic granulomatous disease and Job syndrome.

Which autoimmune disease affects the central nervous system?
1
Uveitis
2
Celiac disease
3
Multiple sclerosis
4
Goodpasture syndrome

3 Multiple sclerosis is an autoimmune disease that affects the central nervous system. Uveitis is an autoimmune disease that affects the eyes. Celiac disease is an autoimmune disease that affects the gastrointestinal System. Goodpasture syndrome is an autoimmune disease that affects the kidneys.

A nurse is teaching parents of toddlers about why children receiving specific medications should not receive varicella vaccines. Which medication will be included in the discussion?
1
Insulin
2
Steroids
3
Antibiotics
4
Anticonvulsants

2 Steroids have an immunosuppressive effect. It is thought that resistance to certain viral diseases, including varicella, is greatly decreased when a child takes steroids regularly. There is no known correlation between varicella and insulin. Because varicella is a viral disease, antibiotics will have no effect. There is no known correlation between varicella and anticonvulsants.

Which leukocytes should the nurse include when teaching about antibody-mediated immunity? Select all that apply.
1
Monocyte
2
Memory cell
3
Helper T cell
4
B-lymphocyte
5
Cytotoxic T cell

2, 4 Memory cells and B-lymphocytes are involved in antibody-mediated immunity. Monocytes are involved in inflammation. Helper T cells and cytotoxic T cells are involved in cell-mediated immunity.

Which are examples of a type IV hypersensitivity reaction? Select all that apply.
1
Poison ivy allergic reaction
2
Sarcoidosis
3
Myasthenia gravis
4
Rheumatoid arthritis
5
Systemic lupus erythematosus

1, 2 Sarcoidosis and poison ivy reactions are examples of type IV hypersensitivity reactions. In type IV hypersensitivity, the inflammation is caused by a reaction of sensitized T cells with the antigen and the resultant activation of macrophages due to lymphokine release. Myasthenia gravis is an example of a type II or cytotoxic hypersensitivity reaction. Rheumatoid arthritis and systemic lupus erythematosus are examples of type III immune complex-mediated reactions.

Which client is most likely to develop IgE antibodies?
1
A client with pollen allergy
2
A client undergoing a poison ivy reaction
3
A client with bacterial infection
4
A client undergoing a blood transfusion

1 A client with a pollen allergy develops IgE antibodies that may result in an anaphylactic reaction. A client with poison ivy develops delayed hypersensitivity, which is mediated by T lymphocytes. A client with a bacterial infection develops IgG and IgM antibodies. A client undergoing blood transfusion may develop IgG and IgM type II hypersensitivity reactions.

Which bacteria colonies are commonly found in a client’s large intestine?
1
Escherichia coli
2
Neisseria gonorrhoeae
3
Staphylococcus aureus
4
Haemophilus influenzae

1 Escherichia coli are bacteria that are part of the normal flora in the large intestine. Neisseria gonorrhoeae causes gonorrhea and pelvic inflammatory disease. Staphylococcus aureus secretes toxins that damage cells and causes skin infections, pneumonia, urinary tract infections, acute osteomyelitis, and toxic shock syndrome. Haemophilus influenzae causes nasopharyngitis, meningitis, and pneumonia.

Which preparations use toxoids but not live viruses? Select all that apply.
1
Rotarix
2
Varivax
3
M-M-R II
4
PEDIARIX
5
DAPTACEL

4, 5 PEDIARIX consists of diphtheria and tetanus toxoids plus inactivated bacterial components of pertussis, inactive viral antigen of hepatitis B, and inactivated poliovirus vaccine. DAPTACEL is a preparation consisting of toxoids plus inactive bacterial and viral components of diphtheria and tetanus toxoids and acellular pertussis vaccine. Rotarix, Varivax, and M-M-R II are preparations containing live viruses.

The nurse is reviewing the laboratory report of four clients. Which does the nurse suspect to have acquired immunodeficiency syndrome (AIDS)?
1
Client A – 3000 cells/mm
2
Client B – 5000 cells/mm
3
Client C – 7000 cells/mm
4
Client D – 9000 cells/mm

1 The normal lymphocyte count is between 5000 and 10,000 cells/mm 3. A client with AIDS is leukopenic and has a lymphocyte count less than 3500 cells/mm 3. Therefore, client A has AIDS. Clients B, C, and D have normal lymphocyte counts.

Which diseases are caused by viruses? Select all that apply.
1
Mumps
2
Tetanus
3
Measles
4
Hepatitis B
5
Diphtheria

1, 3, 4 Mumps is a swelling of the parotid glands caused by a virus. Measles is a highly contagious viral disease characterized by rash and high fever. Hepatitis B (a serious liver infection) is caused by a virus. Tetanus is caused by Clostridium tetani and diphtheria is caused by Corynebacterium diphtheriae. Both are gram-positive bacilli bacterial strains.

Which cytokine increases growth and maturation of myeloid stem cells?
1
Interleukin-2
2
Thrombopoietin
3
Granulocyte colony-stimulating factor
4
Granulocyte-macrophage colony-stimulating factor

4 Granulocyte-macrophage colony-stimulating factor is a cytokine that increases growth and maturation of myeloid stem cells. Interleukin-2 is a cytokine that increases growth and differentiation of T-lymphocytes. Thrombopoietin is a cytokine that increases growth and differentiation of platelets. Granulocyte colony-stimulating factor is a cytokine that increases numbers and maturity of neutrophils.

The mother of an 8-year-old child with the diagnosis of acute poststreptococcal glomerulonephritis (APSGN) is concerned that a 4-year-old sibling may also have the disorder. What does the nurse recall when preparing to explain the cause of the disease process?
1
A systemic infection causing clots in the small renal tubules
2
A factor that is unknown and therefore is difficult to prevent
3
An immune complex disorder occurring after a group A β-hemolytic Streptococcus infection
4
An autosomal recessive trait, meaning that there is an increased probability that a sibling will also have the disease

3 The β-hemolytic Streptococcus immune complex becomes trapped in the glomerular capillary loop, causing acute poststreptococcal glomerulonephritis. APSGN is usually precipitated by a localized pharyngitis. Clots do not form in the small renal tubules with APSGN. Prevention depends on treating an individual with a group A β-hemolytic Streptococcus infection with antibiotics to eliminate the organism before an immune response can occur. APSGN is an acquired, not an inherited, disorder.

A nurse is providing dietary teaching for a client with celiac disease. Which foods should the nurse teach the client to avoid when following a gluten-free diet? Select all that apply.
1
Rye
2
Oats
3
Rice
4
Corn
5
Wheat

1, 2, 5 Rye, oats, and wheat should be avoided because they are irritating to the gastrointestinal mucosa. Gluten is not found in rice or corn; therefore, these items do not have to be avoided.

Which cytokine is used to treat multiple sclerosis?
1
β-Interferon
2
Interleukin-2
3
Erythropoietin
4
Colony-stimulating factor

1 β-Interferon is a cytokine used to treat multiple sclerosis. Interleukin-2 is used to treat metastatic melanoma. Erythropoietin is a cytokine used to treat anemia related to chemotherapy. Colony-stimulating factor is a cytokine used to treat chemotherapy-induced neutropenia.

A client reports disturbed sleep due to itching caused by an allergy. Which medication would be prescribed to help the client sleep well and treat the allergic symptoms?
1
Cetirizine
2
Fexofenadine
3
Desloratadine
4
Chlorpheniramine

4 Chlorpheniramine [1] [2] is an antihistamine that helps to manage allergic symptoms by preventing vasodilation and decreasing allergic symptoms. Sedation is a side effect of chlorpheniramine; therefore this drug is prescribed to clients experiencing sleep issues due to allergic symptoms. Cetirizine effectively blocks histamine from binding to receptors and has less sedating potential. Fexofenadine and desloratadine are also less sedating antihistamine drugs.

Which type of hypersensitivity reaction will occur when the client’s T cytotoxic cells are involved as the mediators of injury?
1
Type I
2
Type II
3
Type III
4
Type IV

4 Type IV hypersensitivity reaction will occur when the T cytotoxic cells are involved as the mediators of injury. Type I IgE-mediated reaction will occur when histamine is involved as the mediators of injury. Type II cytotoxic reaction will occur when complement lysis is the mediator of injury. Type III immune complex reaction will occur when neutrophils are involved as the mediators of injury.

A client has a kidney transplant. The nurse should monitor for which assessment findings associated with rejection of the transplant? Select all that apply.
1
Fever
2
Oliguria
3
Jaundice
4
Polydipsia
5
Weight gain

1, 2, 5 Fever is a characteristic of the systemic inflammatory response to the antigen (transplanted kidney). Oliguria or anuria occurs when the transplanted kidney is rejected and fails to function. Weight gain can occur from fluid retention when the transplanted kidney fails to function or as a result of steroid therapy; this response must be assessed further. Jaundice is unrelated to rejection. Polydipsia is associated with diabetes mellitus; it is not a clinical manifestation of rejection.

What does the nurse explain to a client that a positive diagnosis for human immunodeficiency virus (HIV) infection is based on?
1
Performance of high-risk sexual behaviors
2
Evidence of extreme weight loss and high fever
3
Identification of an associated opportunistic infection
4
Positive enzyme-linked immunosorbent assay (ELISA) and Western blot tests

4 Positive ELISA and Western blot tests confirm the presence of HIV antibodies that occur in response to the presence of the HIV. Performance of high-risk sexual behaviors places someone at risk but does not constitute a positive diagnosis. Evidence of extreme weight loss and high fever do not confirm the presence of HIV; these adaptations are related to many disorders, not just HIV infection. The diagnosis of just an opportunistic infection is not sufficient to confirm the diagnosis of HIV. An opportunistic infection (included in the Centers for Disease Control and Prevention surveillance case definition for acquired immunodeficiency syndrome [AIDS]) in the presence of HIV antibodies indicates that the individual has AIDS.

Which organ-specific autoimmune disorder is associated with a client’s kidney?
1
Graves’ disease
2
Addison’s disease
3
Goodpasture syndrome
4
Guillain-Barré syndrome

3 Goodpasture syndrome is an autoimmune disorder associated with the client’s kidney. Graves’ disease and Addison’s disease are autoimmune disorders associated with the endocrine system. Guillain-Barré syndrome is an autoimmune disorder associated with the central nervous system.

Which leukocyte releases vasoactive amines during a client’s allergic reactions?
1
Neutrophil
2
Monocyte
3
Eosinophil
4
Macrophage

3 Eosinophils release vasoactive amines during allergic reactions to limit the extent of the allergic reactions. Neutrophils are phagocytes and increase in inflammation and infection. Monocytes are involved in the destruction of bacteria and cellular debris. Macrophages are involved in nonspecific recognition of foreign protein and microorganisms.

A nurse is caring for a 13-year-old child who has an external fixation device on the leg. What is the nurse’s priority goal when providing pin care?
1
Easing pain
2
Minimizing scarring
3
Preventing infection
4
Preventing skin breakdown

3 Pin sites provide a direct avenue for organisms into the bone. Pin care will not ease pain. Some scarring will occur at the pin insertion site regardless of pin site care. Skin has a tendency to grow around the pin, rather than break down, as long as infection is prevented.

Which hypersensitivity reaction may occur in a newborn with hemolytic disease?
1
Type I
2
Type II
3
Type III
4
Type IV

2 Hemolytic disease in a pregnant woman may result in erythroblastosis fetalis, a type II hypersensitivity reaction. Type I reactions involve immunoglobulin E (IgE)-mediated reactions such as anaphylaxis and wheal-and-flare reactions. Type III reactions are immune complex reactions such as rheumatoid arthritis or systemic lupus erythematosus. Type IV reactions are delayed hypersensitivity reactions such as contact dermatitis.

An injured client with an open wound is brought to the hospital. The doctor asks the nurse to administer a tetanus toxoid injection. Which standard of the nursing practice as defined by the American Nurses Association does the nurse follow?
1
Diagnosis
2
Evaluation
3
Assessment
4
Implementation

4 The nurse will administer the tetanus as per the doctor’s regime. The American Nurses Association identifies this standard of nursing practice as implementation. Diagnosis refers to analysis of the client’s biological and psychosocial data to find out the relevant issues and problems. Evaluation is the procedure of assessing the desired outcomes of treatment. Assessment is done at the very beginning when the nurse collects the data about the client to make an accurate diagnosis.

Which autoantigens are responsible for the development of Crohn’s disease?
1
Crypt epithelial cells
2
Thyroid cell surface
3
Basement membranes of the lungs
4
Basement membranes of the glomeruli

1 Crypt epithelial cells are considered to be the autoantigens responsible for Crohn’s disease. Thyroid cell surfaces are autoantigens responsible for Hashimoto’s thyroiditis. The pulmonary and glomerular basement membranes act as autoantigens responsible for Goodpasture syndrome.

A mother with the diagnosis of acquired immunodeficiency syndrome (AIDS) states that she has been caring for her baby even though she has not been feeling well. What important information should the nurse determine?
1
If she has kissed the baby
2
If the baby is breast-feeding
3
When the baby last received antibiotics
4
How long she has been caring for the baby

2 Epidemiologic evidence has identified breast milk as a source of human immunodeficiency virus (HIV) transmission. Kissing is not believed to transmit HIV. When the baby last received antibiotics is unrelated to transmission of HIV. HIV transmission does not occur from contact associated with caring for a newborn.

A client is admitted to the hospital with a diagnosis of acute Guillain-Barré syndrome. Which assessment is priority?
1
Urinary output
2
Sensation to touch
3
Neurologic status
4
Respiratory exchange

4 The respiratory center in the medulla oblongata can be affected with acute Guillain-Barré syndrome because the ascending paralysis can reach the diaphragm, leading to death from respiratory failure. Although urinary output, sensation to touch, and neurologic status are important, none of them are the priority.

Which autoimmune disease is directly related to the client’s central nervous system?
1
Rheumatic fever
2
Multiple sclerosis
3
Myasthenia gravis
4
Goodpasture syndrome

2 Multiple sclerosis is a central nervous system-specific autoimmune disease. Rheumatic fever is related to the heart. Myasthenia gravis is a muscle-related autoimmune disease. Goodpasture syndrome is a kidney-related autoimmune disease.

A client who has acquired human immunodeficiency syndrome (HIV) develops bacterial pneumonia. On admission to the emergency department, the client’s PaO 2 is 80 mm Hg. When the arterial blood gases are drawn again, the level is determined to be 65 mm Hg. What should the nurse do first?
1
Prepare to intubate the client.
2
Increase the oxygen flow rate per facility protocol.
3
Decrease the tension of oxygen in the plasma.
4
Have the arterial blood gases redone to verify accuracy.

2 This decrease in PaO 2 indicates respiratory failure; it warrants immediate medical evaluation. Most facilities have a protocol to increase the oxygen flow rate to keep oxygen saturation greater than 92%. The client PaO 2 of 65 mm Hg is not severe enough to intubate the client without first increasing flow rate to determine if the client improves. Decreasing the tension of oxygen in the plasma is inappropriate and will compound the problem. The PaO 2 is a measure of the pressure (tension) of oxygen in the plasma; this level is decreased in individuals who have perfusion difficulties, such as those with pneumonia. Having the arterial blood gases redone to verify accuracy is negligent and dangerous; a falling PaO 2 level is a serious indication of worsening pulmonary status and must be addressed immediately. Drawing another blood sample and waiting for results will take too long.

Which client organ is protected by microglial cells?
1
Lung
2
Liver
3
Brain
4
Kidney

3 Microglial cells are macrophages present in the brain. The lungs are protected by alveolar macrophages. The liver is protected by Kupffer cells. Mesangial cells are present in the kidneys.

Which type of cytokine is used to treat anemia related to chronic kidney disease?
1
α-Interferon
2
Interleukin-2
3
Interleukin-11
4
Erythropoietin

4 Erythropoietin is used to treat anemia related to chronic kidney disease. α-Interferon is used to treat hairy cell leukemia or malignant melanoma. Interleukin-2 is used to treat metastatic renal carcinoma. Interleukin-11 is used to prevent thrombocytopenia after chemotherapy.

Which type of immunity is acquired through the transfer of colostrum from the mother to the child?
1
Natural active immunity
2
Artificial active immunity
3
Natural passive immunity
4
Artificial passive immunity

3 Natural passive immunity is acquired through the transfer of colostrum from the mother to the child. Natural active immunity is acquired when there is a natural contact with an antigen through a clinical infection. Artificial active immunity is acquired through immunization with an antigen. Artificial passive immunity is acquired by injecting serum from an immune human.

A nurse is caring for a client who is human immunodeficiency virus (HIV) positive. Which complication associated with this diagnosis is most important for the nurse to teach prevention strategies?
1
Infection
2
Depression
3
Social isolation
4
Kaposi sarcoma

1 The client has a weakened immune response. Instructions regarding rest, nutrition, and avoidance of unnecessary exposure to people with infections help reduce the risk for infection. Clients can be taught cognitive strategies to cope with depression, but the strategies will not prevent depression. The client may experience social isolation as a result of society’s fears and misconceptions; these are beyond the client’s control. Although Kaposi sarcoma is related to HIV infection, there are no specific measures to prevent its occurrence.

Which type of hypersensitivity reaction is associated with rheumatoid arthritis?
1
Delayed
2
Cytotoxic
3
IgE-mediated
4
Immune-complex

4 Rheumatoid arthritis is an autoimmune disorder associated with an immune-complex type of hypersensitivity reaction. Contact dermatitis caused by poison ivy is associated with a delayed type of hypersensitivity reaction. Goodpasture’s syndrome is associated with a cytotoxic type of hypersensitivity reaction. Asthma is associated with an IgE-mediated type of hypersensitivity reaction.

Which sexually transmitted disease is caused by a virus?
1
Syphilis
2
Gonorrhea
3
Genital warts
4
Chlamydial infection

3 Genital warts are caused by a sexually transmitted virus. Bacteria cause syphilis, gonorrhea, and chlamydial infections.

Which process does the IgD immunoglobulin support?
1
Manifestation of allergic reactions
2
Protection of the body’s mucous surfaces
3
Differentiation of the B-lymphocytes
4
Provision of the primary immune response

3 IgD is present on the lymphocyte surface; this immunoglobulin differentiates B-lymphocytes. IgE causes symptoms of allergic reactions by adhering to mast cells and basophils. IgE also helps to defend the body against parasitic infections. IgA lines the mucous membranes and protects the body surfaces. IgM provides the primary immune response.

Which type of immune preparation, made from donated blood, contains antibodies that provide passive immunity?
1
Toxoid
2
Killed vaccine
3
Live attenuated vaccine
4
Specific immune globulin

4 Specific immune globulins contain a high concentration of antibodies directed at specific antigens. Toxoid vaccines contain a bacterial toxin that has been changed to a nontoxic form. Killed vaccines contain killed microbes or isolated microbes. Live attenuated vaccines are composed of live microbes that have been weakened or rendered completely avirulent.

A client who abused intravenous drugs was diagnosed with the human immunodeficiency virus (HIV) several years ago. What does the nurse explain to the client regarding the diagnostic criterion for acquired immunodeficiency syndrome (AIDS)?
1
Contracts HIV-specific antibodies
2
Develops an acute retroviral syndrome
3
Is capable of transmitting the virus to others
4
Has a CD4+T-cell lymphocyte level of less than 200 cells/µL (60%)

4 AIDS is diagnosed when an individual with human immunodeficiency virus (HIV) develops one of the following: a CD4+T-cell lymphocyte level of less than 200 cells/µL (60%), wasting syndrome, dementia, one of the listed opportunistic cancers (e.g., Kaposi sarcoma [KS], Burkitt lymphoma), or one of the listed opportunistic infections (e.g., Pneumocystis jiroveci pneumonia, Mycobacterium tuberculosis). The development of HIV-specific antibodies (seroconversion), accompanied by acute retroviral syndrome (flulike syndrome with fever, swollen lymph glands, headache, malaise, nausea, diarrhea, diffuse rash, joint and muscle pain) 1 to 3 weeks after exposure to HIV reflects acquisition of the virus, not the development of AIDS. A client who is HIV positive is capable of transmitting the virus with or without the diagnosis of AIDS.

A father asks the nurse about the immunization schedule for his 15-month-old toddler, who is being treated for acute lymphoid leukemia. What vaccine is contraindicated for a child undergoing chemotherapy?
1
Hib (influenza)
2
Hep B (hepatitis B)
3
MMR (measles, mumps, rubella)
4
DTaP (diphtheria, tetanus, acellular pertussis)

3 The MMR vaccine contains attenuated live virus and should not be administered to a child undergoing chemotherapy because of the compromise of the child’s immune system. There are no contraindications to administering the Hib vaccine, Hep B vaccine, or DTaP vaccine to a child who is immunosuppressed.

What are the symptoms of tuberculosis? Select all that apply.
1
Fatigue
2
Nausea
3
Weight gain
4
Low-grade fever
5
Increased appetite

1, 2, 4 Tuberculosis is an infectious respiratory disease caused by Mycobacterium tuberculosis. The symptoms of tuberculosis are fatigue, nausea, low-grade fever, weight loss, and anorexia.

A child is found to be allergic to dust. The nurse is preparing a teaching plan for the parents. What should the nurse include in the plan?
1
Housework must be done by professional housecleaners.
2
Damp-dusting the house will help limit dust particles in the air.
3
The condition must be accepted because dust cannot be limited.
4
The house must be redecorated because the environment must be dust free.

2 Although dust cannot be avoided completely, use of a damp cloth helps eliminate the quantity of airborne particles that might be inhaled. Hiring professional housecleaners is unnecessary and unrealistic. There are ways to limit the quantity of airborne particles. Redecorating will not eliminate dust; it is part of our environment

A nurse has just administered an immunization injection to a 2-month-old infant. What instructions should the nurse give the parent if the infant has a reaction?
1
Give aspirin for pain; if swelling at the injection site develops, call the healthcare provider.
2
Apply heat to the injection site for the first day after the injection; apply ice if the arm is inflamed.
3
Give acetaminophen for fever; call the healthcare provider if the child exhibits marked drowsiness or seizures.
4
Apply ice to the injection site if soreness develops; call the healthcare provider if the child comes down with a fever.

3 Fever is a common reaction to immunizations, and acetaminophen may be given to minimize discomfort. A central nervous system reaction is rare and requires notification of the healthcare provider. Aspirin should not be given to infants and children because it is linked to Reye syndrome. Infants do not tolerate the application of ice, which will increase discomfort. Fever is a common reaction to the immunizations; it is not necessary to notify the healthcare provider.

Why would a client with acquired immunodeficiency syndrome (AIDS) be prescribed diphenoxylate hydrochloride?
1
To manage pain
2
To manage diarrhea
3
To manage candidal esophagitis
4
To manage behavioral problems

2 Diphenoxylate hydrochloride is an antidiarrheal drug prescribed to clients with AIDS to manage frequent diarrhea experienced by a client with AIDS. Opioid analgesics such as tramadol are used to manage pain. Ketoconazole can be used to treat candidal esophagitis associated with AIDS. Behavioral problems are managed with psychotropic drugs.

A client who underwent a physical examination reports itching after 2 days. Which condition should the nurse suspect?
1
Eczema
2
Hypersensitivity
3
Contact dermatitis
4
Anaphylactic shock

3 A client who is allergic to latex may experience an allergy after a physical examination with latex gloves. Itching is one of the clinical signs of latex allergy. Contact dermatitis is a delayed immune response that occurs 12 to 48 hours after exposure. Eczema is a skin condition that can be worsened with excessive drying. Hypersensitivity is an immediate allergic reaction that occurs due to chemicals that are used to make gloves. Anaphylactic shock is also an immediate allergic reaction that occurs due to natural rubber latex.

Which virus can cause encephalitis in adults and children?
1
Rubella virus
2
Parvovirus
3
Rotaviruses
4
West Nile virus

4 The West Nile virus causes encephalitis. German measles is caused by rubella. Gastroenteritis is caused by parvovirus. Rotavirus also causes gastroenteritis.

What is the function of a client’s natural killer cells?
1
Secrete immunoglobulins in response to the presence of a specific antigen
2
Heighten selectively and destroy non-self cells, including virally infected cells
3
Enhance immune activity through secretion of various factors, cytokines, and lymphokines
4
Attack non-selectively on non-self cells, especially mutated and malignant cells

4 Natural killer cells attack non-selectively on non-self cells, especially body cells that have undergone mutation and become malignant. Plasma cells secrete immunoglobulins in response to the presence of a specific antigen. Cytotoxic T-cells attack selectively and destroy non-self cells, including virally infected cells. Helper T-cells enhance immune activity through secretion of various factors, cytokines, and lymphokines.

What finding in the client is a sign of allergic rhinitis?
1
Presence of high-grade fever
2
Reduced breathing through the mouth
3
Presence of pinkish nasal discharge
4
Reduced transillumination on the skin over the sinuses

4 Reduced transillumination on the skin overlying the sinuses indicates allergic rhinitis. This effect is caused by the sinuses becoming inflamed and blocked with thick mucoid secretions. Generally, fever does not accompany allergic rhinitis unless the client develops a secondary infection. In allergic rhinitis, the client is unable to breathe through the nose because it gets stuffy and blocked. Instead the client will resort to mouth breathing. Clients with allergic rhinitis will have clear or white nasal discharge.

According to the Healthcare Personnel Vaccination Recommendations, what meningococcal conjugate vaccine dose should a nurse administer to a 12-year-old with an HIV infection?
1
Single initial dose and a booster dose 3 years later
2
Single initial dose and a booster dose 5 years later
3
Single initial dose and a booster dose 7 years later
4
Two initial doses and a booster dose at 16 years old

4 A 12-year-old with HIV would require two primary meningococcal conjugate vaccine delivered two months apart initially and a booster dose at the age of 16 years old. The client would require two initial doses, not a single initial dose, and a booster at 16 years old, not 3, 5, or 7 years later.

A client with localized redness and swelling due to a bee sting reports intense local pain, a burning sensation, and itching. What would be the most appropriate nursing action?
1
Applying cold compresses to the affected area
2
Ensuring the client keeps the skin clean and dry
3
Monitoring for neurological and cardiac symptoms
4
Advising the client to launder all clothes with bleach

1 A client with a bee sting may have localized redness, swelling, pain, and itching due to an allergic reaction. The nurse should apply cold compresses to the affected area to reduce the pain in the client. A client with Candida albicans infection should keep his or her skin clean and dry to prevent further fungal infections. A client with a Borrelia burgdorferi infection may suffer from cardiac, arthritic, and neurologic manifestations. Therefore the nurse has to monitor for these symptoms. Direct contact may transmit a Sarcoptes scabiei infection; the nurse should make sure that the client’s clothes are bleached to prevent the transmission of the infection.

The nurse suspects that a client with inhalation anthrax is in the fulminant stage of the disease. Which symptom supports the nurse’s conclusion?
1
Fever
2
Dry cough
3
Hematemesis
4
Mild chest pain

3 Inhalation anthrax is a bacterial infection caused by Bacillus anthracis. This disease has two stages of illness, the prodromal stage and the fulminant stage. The symptom of the fulminant stage is hematemesis. The symptoms of the prodromal stage are fever, dry cough, and mild chest pain.

What type of hypersensitivity reaction is the cause of systemic lupus erythematosus?
1
Type I
2
Type II
3
Type III
4
Type IV
Systemic lupus erythematosus is an example of an immune complex-mediated, or type III, hypersensitive reaction.

3 Anaphylaxis is an example of a type I or immediate hypersensitive reaction. Cytotoxic or type II hypersensitive reactions can result in conditions such as myasthenia gravis and Goodpasture syndrome. Graft rejection and sarcoidosis are conditions that are caused by delayed or type IV hypersensitivity reactions.

A nurse is teaching a client about human immunodeficiency virus (HIV). What are the various ways HIV is transmitted? Select all that apply.
1
Mosquito bites
2
Sharing syringe needles
3
Breastfeeding a newborn
4
Kissing the infected partner
5
Anal intercourse

2, 3, 5 Fluids such as blood and semen are highly concentrated with HIV. HIV may be transmitted parenterally by sharing needles and postnatally through breast milk. HIV may also be transmitted through anal intercourse. HIV is not transmitted by mosquito bites or kissing.

Which viral infection will cause the nurse to observe for warts?
1
Pox virus
2
Rhabdovirus
3
Epstein-Barr virus
4
Papillomavirus

4 Warts are caused by papillomavirus. Pox viruses cause smallpox. Rhabdovirus causes rabies. Epstein-Barr causes mononucleosis and Burkitt’s lymphoma.

A nurse is monitoring a client who is having a computed tomography (CT) scan of the brain with contrast. Which response indicates that the client is having an untoward reaction to the contrast medium?
1
Pelvic warmth
2
Feeling flushed
3
Shortness of breath
4
Salty taste in the mouth

3 An untoward response to the iodinated dye used as a contrast is anaphylaxis, a life-threatening allergic response. Anaphylaxis is manifested by respiratory distress, hypotension, and shock; counteractive measures must be instituted. A feeling of warmth or flushing is an expected minor side effect. A salty taste is an expected minor side effect.

What causes medications used to treat AIDS to become ineffective?
1
Taking the medications 90% of the time
2
Missing doses of the prescribed medications
3
Taking medications from different classifications
4
Developing immune reconstitution inflammatory syndrome (IRIS)

2 The most important reason for the development of drug resistance in the treatment of AIDS is missing doses of drugs. When doses are missed, the blood drug concentrations become lower than what is needed to inhibit viral replication. The virus replicates and produces new particles that are resistant to the drugs. Taking the medications 90% of the time prevents medications from becoming ineffective. Taking medications from different classes prevents the drugs from becoming ineffective. Immune reconstitution inflammatory syndrome (IRIS) occurs when T-cells rebound with medication therapy and become aware of opportunistic infections.

A nurse is caring for a client with acquired immunodeficiency syndrome (AIDS). What precautions should the nurse take when caring for this client?
1
Use standard precautions.
2
Employ airborne precautions.
3
Plan interventions to limit direct contact.
4
Discourage long visits from family members.

1 The Centers for Disease Control and Prevention (Canada: Public Health Agency of Canada) states that standard precautions should be used for all clients; these precautions include wearing of gloves, gown, mask, and goggles when there is risk for exposure to blood or body secretions. There is no indication that airborne precautions are necessary. Planning interventions to limit direct contact and discouraging long visits from family members will unnecessarily isolate the client.

Which is the first antibody formed after exposure to an antigen?
1
IgA
2
IgE
3
IgG
4
IgM

4 IgM (immunoglobulin M) is the first antibody formed by a newly sensitized B-lymphocyte plasma cell. IgA has very low circulating levels and is responsible for preventing infection in the upper and lower respiratory tracts, and the gastrointestinal and genitourinary tracts. IgE has variable concentrations in the blood and is associated with antibody-mediated hypersensitivity reactions. IgG is heavily expressed on second and subsequent exposures to antigens to provide sustained, long-term immunity against invading microorganisms.

The nurse administers an initial dose of Haemophilus influenzae type b (Hib) vaccine to a 2-month-old infant. When should the nurse administer the final dose of the vaccine to the infant?
1
6-8 months of age
2
8-10 months of age
3
12-15 months of age
4
16-18 months of age

3 The Haemophilus influenzae type b (Hib) vaccine is administered in four doses, finishing at the age of 12-15 months. Following the first dose at 2 months, the second is administered at 4 months, and the third at 6 months. For the final dose, 6-8 months and 8-10 months would be too soon; 16-18 months would be too late.

Which conditions result in humoral immunity? Select all that apply.
1
Tuberculosis
2
Atopic diseases
3
Bacterial infection
4
Anaphylactic shock
5
Contact dermatitis

2, 3, 4 Atopic diseases, bacterial infections, and anaphylactic shock are disease conditions that trigger humoral immunity. Tuberculosis and contact dermatitis result in cell-mediated immunity.

What is the mechanism of action of norepinephrine in managing anaphylaxis?
1
Norepinephrine blocks the effects of histamine
2
Norepinephrine inhibits the degranulation of mast cells
3
Norepinephrine increases blood pressure and cardiac output
4
Norepinephrine rapidly stimulates alpha- and beta-adrenergic receptors

3 Norepinephrine is a vasopressor that elevates the blood pressure and cardiac output in clients suffering from anaphylactic reactions. Diphenhydramine HCL blocks the effects of histamine on various organs. Corticosteroids such as dexamethasone prevent the degranulation of mast cells. Epinephrine works by rapidly stimulating alpha- and beta-adrenergic receptors.

After assessing an older client’s medical report, the nurse finds that the client is at an increased risk for bacterial and fungal infections. Which change in immune function may have occurred?
1
Decline in natural bodies
2
Reduction of neutrophil function
3
Decrease in circulating T-lymphocytes
4
Reduction of colony-forming B-lymphocytes

3 A decrease in circulating T-lymphocytes occurs with cell-mediated immunity, resulting in an increased risk of bacterial and fungal infections. A client would need booster shots for old vaccinations and immunizations when there is a decline in natural antibodies. A reduced neutrophil function may be an implication when neutrophil function is decreased. The older adult should receive immunizations, such as flu shots, when the number of colony-forming B-lymphocytes is diminished.

Which diseases can be transmitted from client to client by droplet infection? Select all that apply.
1
Scabies
2
Shingles
3
Measles
4
Pertussis
5
Diphtheria

4, 5 Pertussis and diphtheria are infectious diseases that are known to be transmitted by droplet infection. Shingles and measles are infectious diseases that are known to be transmitted by air. Scabies is an infectious disease that is transmitted by direct contact.

A client has undergone scratch testing but the causative allergen is yet to be identified. What would be the next step to confirm a strongly suspected allergen?
1
Administer allergy shots
2
Begin intradermal testing
3
Request that a client take an oral food challenge
4
Begin radioallergosorbent testing

2 An intradermal test is administered when a strongly suspected allergen tests negative with the scratch test. A testing dose of sera is injected intradermally into the arm and any signs of allergy are observed to confirm the allergen. Allergy shots are a therapeutic method of reducing sensitivity to a known allergen when exposure cannot be avoided. An oral food challenge is done to confirm an ingested allergen if skin testing completely fails. Radioallergosorbent testing is useful to measure immunoglobulin E levels to ascertain the presence of an allergic reaction.

Which medications act by binding with integrase enzyme and prevent human immunodeficiency virus (HIV) from incorporating its genetic material into the client’s cell? Select all that apply.
1
Ritonavir
2
Nelfinavir
3
Tenofovir
4
Raltegravir
5
Elvitegravir

4, 5 Raltegravir and elvitegravir are integrase inhibitors. They act by binding with integrase enzyme and prevent HIV from incorporating its genetic material into the client’s cell. Ritonavir and nelfinavir are protease inhibitors. They act by preventing the protease enzyme from cutting HIV proteins into the proper lengths needed to allow viable virions to assemble and bud out from the cell membrane. Tenofovir is a nucleotide reverse transcriptase inhibitor. It acts by combining with reverse transcriptase enzyme to block the process needed to convert HIV ribose nucleic acid into HIV deoxyribose nucleic acid.

What functions of leukocytes are involved in inflammation? Select all that apply.
1
Destruction of bacteria and cellular debris
2
Selective attack and destruction of non-self cells
3
Release of vasoactive amines during allergic reactions
4
Secretion of immunoglobulins in response to a specific antigen
5
Enhancement of immune activity through secretion of various factors, cytokines, and lymphokines

1, 3 Leukocytes such as monocytes and eosinophils are involved in inflammation. Their functions include the destruction of bacteria and cellular debris and the release of vasoactive amines during allergic reactions to limit these reactions. Helper/inducer T-cells and cytotoxic cells selectively attack and destroy non-self cells and secrete immunoglobulins in response to the presence of a specific antigen. B-lymphocytes, or plasma cells, secrete immunoglobulins in response to the presence of a specific antigen. Helper/inducer T-cells are involved in cell-mediated immunity, enhancing immune activity through the secretion of various factors, cytokines, and lymphokines.

When taking the blood pressure of a client who has acquired immunodeficiency syndrome (AIDS), what must the nurse do?
1
Don clean gloves.
2
Use barrier techniques.
3
Put on a mask and gown.
4
Wash hands thoroughly.

4 Because this procedure does not involve contact with blood or secretions, additional protection other than washing the hands thoroughly is not indicated. Donning clean gloves and using barrier techniques are necessary only when there is risk of contact with blood or body fluid. A mask and gown are indicated only if there is a danger of secretions or blood splattering on the nurse (for example, during suctioning).

The nurse educates a client on decreasing the risk of developing antibiotic-resistant infections. Which statement made by the nurse will be most significant?
1
"Wash your hands frequently."
2
"Do not skip any dose of your antibiotics."
3
"Save the unfinished antibiotics for later use."
4
"Stop taking the antibiotics when you feel better."

2 Antibiotic-resistant infection develops when the hardiest bacteria survive and multiply. This may happen when a client stops taking an entire course of antibiotics, which leads to infections that are resistant to many antibiotics. Therefore a client should not skip any dose of an antibiotic. Hand washing is required to prevent infections; it is not related to antibiotic-resistant infections. Antibiotics should not be stopped even if the client has started feeling better; the full course of treatment should be taken. Non-compliance in taking the full course of prescribed antibiotics can lead to an antibiotic-resistant infection. It is dangerous to take the unfinished antibiotics at a later time; it may prove fatal if the antibiotics are outdated.

A client who has been living in another country for 10 years is undergoing diagnostic testing to identify the causative organisms of the infection that has been acquired. When caring for this client, what should the nurse recall about active immunity?
1
Protein antigens are formed in the blood to fight invading antibodies.
2
Protein substances are formed within the body to neutralize antigens.
3
Blood antigens are aided by phagocytes in defending the body against pathogens.
4
Sensitized lymphocytes from an immune donor act as antibodies against invading pathogens.

2 Active immunity occurs when the individual’s cells produce antibodies in response to an agent or its products; these antibodies will destroy the agent (antigen) should it enter the body again. Antigens do not fight antibodies; they trigger antibody formation that in turn attacks the antigen. Antigens are foreign substances that enter the body and trigger antibody formation. Sensitized lymphocytes do not act as antibodies.

A nurse who is caring for a 7-year-old child with acute glomerulonephritis assesses the child for cerebral complications. What signs and symptoms indicate cerebral involvement?
1
Headache, drowsiness, and vomiting
2
Generalized edema, anorexia, and restlessness
3
Anuria, temperature higher than 103° F (39.4° C), and confusion
4
Cardiac decompensation, heart rate of 114 beats/min, and vomiting

1 Headache, drowsiness, and vomiting may occur if the blood pressure remains increased and leads to cerebral edema. Drowsiness, not restlessness, will occur; generalized edema and anorexia are not specific to cerebral edema. Although fever and confusion may occur, anuria is not specific to cerebral edema. Although the pulse may be altered and vomiting may occur, cardiac decompensation is not related to cerebral involvement.

A primary healthcare provider has prescribed pyrazinamide to a client with tuberculosis. Which instruction by the nurse will be beneficial to the client? Select all that apply.
1
Avoid drinking alcoholic beverages."
2
"Drink at least 8 ounces of water with the medication."
3
"Your soft contact lenses will be stained permanently."
4
"Darkening of the urine is normal while you are using this drug."
5
"Be sure to report any changes in vision such as diminished color perception."

1, 2 A client undergoing pyrazinamide therapy may require extra fluids to help prevent uric acid formation from precipitating and causing gout or kidney problems. Therefore the client should drink at least 8 ounces of water with the medication. The client should also avoid alcoholic beverages, which could potentiate liver toxicity. Staining is a common problem with rifampin, not pyrazinamide. The client should also report any darkening of urine because this may be a sign of liver toxicity or damage. The client should report any vision changes if he or she is taking etambutol.

A client with a diagnosis of polyarteritis nodosa asks the nurse for information about this disorder. What information should the nurse include in the response?
1
Clients with this disease have an excellent prognosis.
2
The disorder affects males and females in equal numbers.
3
The disorder is considered one of hypersensitivity, and the exact cause is unknown.
4
Clients with this disease have problems with only the kidneys and the retina of the eyes.

3 An autoimmune response plays a role in the development of polyarteritis, although drugs and infections may precipitate it. The disorder often is fatal, usually as a result of heart or renal failure. Men are affected three times more often than women. Arteriolar pathology can affect any organ or system.

When preparing discharge teaching for a client who had a kidney transplant, in addition to a corticosteroid, the nurse expects what other medications to be prescribed to prevent kidney rejection?
1
Furosemide and sirolimus
2
Cefazolin and methotrexate
3
Methylprednisolone and phenytoin
4
Tacrolimus and mycophenolate mofetil

4 Standard triple therapy includes a corticosteroid prednisone (methylprednisolone), an antimetabolite (mycophenolate), and a calcineurin inhibitor (tacrolimus and cyclosporine). Although sirolimus is used for immunosuppression, furosemide is a diuretic. Neither of these medications are immunosupressives. Cefazolin is an antibiotic, and methotrexate is a folic acid antagonist used in cancer chemotherapy. Although methylprednisolone is used for immunosuppression, phenytoin is an antiseizure medication.

Which age-related effects on the immune system are seen in the older client?
Correct1
Increased autoantibodies
2
Increased expression of IL-2 receptors
3
Increased delayed hypersensitivity reaction
4
Increased primary and secondary antibody responses
The effects of aging on the immune system include increased autoantibodies. Expression of IL-2 receptors, delayed hypersensitivity reaction, and primary and secondary antibody responses decrease in older adults because of the effects of aging on the immune system.

Which type of hepatitis virus spreads through contaminated food and water?
Correct1
Hepatitis A virus
2
Hepatitis B virus
3
Hepatitis C virus
4
Hepatitis D virus
Hepatitis A virus spreads through contaminated food and water. Hepatitis B, C, and D viruses spread through contaminated needles, syringes, and blood products.

A nurse is caring for a client with pulmonary tuberculosis. What must the nurse determine before discontinuing airborne precautions?
1
Client no longer is infected.
2
Tuberculin skin test is negative.
Correct3
Sputum is free of acid-fast bacteria.
4
Client’s temperature has returned to normal.
The absence of bacteria in the sputum indicates that the disease can no longer be spread by the airborne route. Treatment is over an extended period; eventually the client may not have an active disease, but still remains infected. Once an individual has been infected, the test will always be positive. The client’s temperature returning to normal is not evidence that the disease cannot be transmitted.

A nurse is caring for a client with pruritic lesions from an IgE-mediated hypersensitivity reaction. Which mediator of injury is involved?
Correct1
Histamine
2
Cytokine
3
Neutrophil
4
Macrophage
Histamine is one of the mediators of injury involving IgE-mediated injury that may cause pruritus. Cytokines are the mediators of injury in delayed hypersensitivity reaction. Neutrophils are involved in immune complex-mediated hypersensitivity reactions. Macrophages in tissues are involved in cytotoxic reactions.

Which type of immunoglobulin is present in tears, saliva, and breast milk?
1
IgE
Correct2
IgA
3
IgG
4
IgM
IgA immunoglobulin is present in tears, saliva, and breast milk. IgE and IgG immunoglobulins are present in plasma and interstitial fluids. IgM immunoglobulin is present in plasma.

Confidence: Nailed It
Stats
8.
Which type of immunity will clients acquire through immunizations with live or killed vaccines?
1
Natural active immunity
Correct2
Artificial active immunity
3
Natural passive immunity
4
Artificial passive immunity
Artificial active immunity is acquired through immunization with live or killed vaccines. Natural active immunity is acquired when there is natural contact with antigens through a clinical infection. Natural passive immunity is acquired through the transfer of colostrums from mother to child. Artificial passive immunity is acquired by injecting serum from an immune human.

The nurse suspects that a client is in the chronic persistent stage of Lyme disease. Which symptoms support the nurse’s suspicion? Select all that apply.
Correct1
Arthritis
2
Dyspnea
3
Dizziness
Correct4
Chronic fatigue
5
Erythema migrans
Lyme disease is a systemic infectious disease caused by the spirochete Borrelia burgdorferi. The symptoms of the chronic persistent stage are arthritis and chronic fatigue. Dyspnea and dizziness are the symptoms of the early disseminated stage. Erythema migrans is observed in the localized stage.

The nurse is counseling a client infected with human immunodeficiency virus (HIV) regarding prevention of HIV transmission. Which statement by the client indicates the nurse needs to follow up?
1
"I should abstain from sexual activity."
Correct2
"I can safely have anal sex without any barriers."
3
"I should get HIV counseling if planning for pregnancy."
4
"I will use condoms while having sexual intercourse."
The client with HIV should use barrier protection when engaging in insertive sexual activity such as anal, oral, and vaginal. Therefore the nurse should follow up to provide the client with the correct information. All the other statements are correct and need no follow up. Abstaining from all sexual activity is a safe way to eliminate the risk of exposure to HIV in semen and vaginal secretions. The client should undergo HIV counseling and routinely offer access to voluntary HIV-antibody testing when planning for pregnancy. The most commonly used barrier is a condom, which allows for protected intercourse.

Which antimicrobial medication acts on susceptible pathogens by inhibiting nucleic acid synthesis?
Incorrect1
Penicillin
Correct2
Actinomycin
3
Erythromycin
4
Cephalosporin
Actinomycin is an antimicrobial medication that acts on susceptible pathogens by inhibiting nucleic acid synthesis. Penicillin acts on susceptible pathogens by inhibiting cell wall synthesis. Erythromycin acts on susceptible pathogens by inhibiting biosynthesis and reproduction. Cephalosporin acts on susceptible pathogens by inhibiting cell wall synthesis.

Which immunoglobulin crosses the placenta?
Incorrect1
IgE
2
IgA
Correct3
IgG
4
IgM
IgG is the only immunoglobulin that crosses the placenta. IgE is found in the plasma and interstitial fluids. IgA lines the mucous membranes and protects body surfaces. IgM is found in plasma; this immunoglobulin activates due to the invasion of ABO blood antigens.

Confidence: Just a Guess
Stats
1.
Before administering the first series of immunizations to a 2-month-old infant, the nurse tells the parent that reactions may occur. What are the characteristics of these reactions?
Correct1
Local or systemic and usually mild
2
Often serious, possibly requiring hospitalization
3
Sometimes causing ulceration at the injection site
4
May be responsible for permanent neurological damage
Mild reactions consist of redness and induration at the injection site, slight fever, and irritability. Serious reactions are not common. Induration at the injection site may occur, but not ulceration. Permanent brain damage is not likely after an immunization.

Confidence: Nailed It
Stats
2.
What is a common characteristic of Sjögren’s syndrome (SS)?
Correct1
Dry eyes
2
Muscle cramping
3
Urinary tract infection
4
Elevated blood pressure
Sjögren’s syndrome (SS) is a group of problems that often appear with other autoimmune disorders. Problems include dry eyes, which are caused by autoimmune destruction of the lacrimal glands. Muscle cramping, urinary tract infection, and elevated blood pressure are not common characteristics of Sjögren’s syndrome (SS).

Which cytokine stimulates the liver to produce fibrinogen and protein C?
1
Interleukin-1
Correct2
Interleukin-6
3
Thrombopoietin
4
Tumor necrosis factor
Interleukin-6 stimulates the liver to produce fibrinogen and protein C. Interleukin-1 stimulates the production of prostaglandins. Thrombopoietin increases the growth and differentiation of platelets. Tumor necrosis factor stimulates delayed hypersensitivity reactions and allergies.

What is the mechanism of action of penicillin?
1
Prevents reproduction of the pathogen
Correct2
Inhibits cell wall synthesis of the pathogen
3
Inhibits nucleic acid synthesis of the pathogen
4
Injures the cytoplasmic membrane of the pathogen
Penicillin is an antimicrobial medication that inhibits cell wall synthesis of the susceptible pathogen. Gentamicin is an antimicrobial medication that prevents the reproduction of the susceptible pathogen. Actinomycin is an antimicrobial medication that inhibits nucleic acid synthesis of the susceptible pathogen. Antifungal agents injure the cytoplasmic membrane of the susceptible pathogen.

Confidence: Nailed It
Stats
6.
Which immunomodulatory is beneficial for the treatment of clients with multiple sclerosis?
1
Interleukin 2
2
Interleukin 11
Correct3
Beta interferon
4
Alpha interferon
Beta interferon is an immunomodulator that is administered in the treatment of multiple sclerosis. Interleukin 11 is used in the prevention of thrombocytopenia after chemotherapy. Interleukin 2 is used for the treatment of metastatic renal cell carcinoma and metastatic melanoma. Alpha interferon is administered for the treatment of hairy cell leukemia, chronic myelogenous leukemia, and malignant melanoma

Which is the most common opportunistic infection in a client infected with human immunodeficiency virus (HIV)?
1
Candidiasis
2
Cryptosporidiosis
3
Toxoplasmosis encephalitis
Correct4
Pneumocystis jiroveci pneumonia
Pneumocystis jiroveci pneumonia (PCP) is the most common opportunistic infection in a client infected with HIV. It causes tachypnea and persistent dry cough. Candidiasis presents in a client infected with AIDS because the immune system can no longer control Candida fungal growth. Cryptosporidiosis, an intestinal infection caused by Cryptosporidium organisms, presents in clients with AIDS as does toxoplasmosis encephalitis, which is caused by Toxoplasma gondii and is acquired through contact with contaminated cat feces or by ingesting infected undercooked meat.

What is the causative organism for syphilis?
Correct1
Treponema pallidum
2
Campylobacter jejuni
3
Trichomonas vaginalis
4
Chlamydia trachomatis
The causative organism for syphilis is Treponema pallidum. Campylobacter jejuni is the causative organism for proctitis. Trichomonas vaginalis is the causative organism for vulvovaginitis. Chlamydia trachomatis is the causative organism for salpingitis.

When is the first dose of Rotarix vaccine administered in infants?
1
Birth to 6 weeks
Correct2
6-12 weeks
3
12-18 weeks
4
18-24 weeks
The Rotarix vaccine prevents rotavirus gastroenteritis and diarrhea-related problems. The vaccination requires two doses. The first dose of Rotarix should be given between 6 and 12 weeks followed by the second dose, which is given four or more weeks after the first dose. Administering the vaccine before the age of 6 weeks is too early, and administering it at 12-18 weeks or 18-24 weeks is too late.

Confidence: Nailed It
Stats
4.
A nurse is planning to provide discharge teaching to the family of a client with acquired immunodeficiency syndrome (AIDS). Which statement should the nurse include in the teaching plan?
Correct1
"Wash used dishes in hot, soapy water."
2
"Let dishes soak in hot water for 24 hours before washing."
3
"You should boil the client’s dishes for 30 minutes after use."
Incorrect4
"Have the client eat from paper plates so they can be discarded."
A person cannot contract human immunodeficiency virus (HIV) by eating from dishes previously used by an individual with AIDS; routine care is adequate. Washing used dishes in hot, soapy water is sufficient care for dishes used by the AIDS client. Dishes do not need to soak for 24 hours before being washed. The client’s dishes do not need to be boiled for 30 minutes after use. Paper plates are fine to use but are not indicated to prevent the spread of AIDS.

Which conditions may result from immunoglobulin IgE antibodies on mast cells reacting with antigens? Select all that apply.
Correct1
Asthma
Correct2
Hay fever
3
Sarcoidosis
4
Myasthenia gravis
Incorrect5
Rheumatoid arthritis
Clinical conditions such as asthma and hay fever are considered type I hypersensitive reactions that are mediated by a reaction between IgE antibodies with antigens. It results in the release of mediators such as histamines. Type IV hypersensitivity reactions such as sarcoidosis results from reactions between sensitized T cells with antigens. Myasthenia gravis results from a type II hypersensitivity reaction that occurs due to an interaction between immunoglobulin IgG and the host cell membrane. Rheumatoid arthritis is a type III hypersensitivity reaction that results from the formation of immune complexes between antigens and antibodies that results in inflammation.

Which is a leukotriene antagonist used to manage and prevent allergic rhinitis?
Correct1
Zileuton
Incorrect2
Ephedrine
3
Scopolamine
4
Cromolyn sodium
Zileuton is a leukotriene antagonist used to manage and prevent allergic rhinitis. Ephedrine is an ingredient in decongestants used to treat allergic rhinitis. Scopolamine is an anticholinergic used to reduce secretions. Cromolyn sodium is a mast cell stabilizing drug used to prevent mast cell membranes from opening when an allergen binds to IgE.

Confidence: Nailed It
Stats
12.
A client’s laboratory report reveals a CD4+ T-cell count of 520 cells/mm 3. According to the Centers for Disease Control and Prevention (CDC), which stage of human immunodeficiency virus (HIV) disease is present in the client?
Correct1
Stage 1
Incorrect2
Stage 2
3
Stage 3
4
Stage 4
According to the CDC, HIV disease is divided into four stages. A client with a CD4+ T-cell count of greater than 500 cells/mm 3 is in the first stage of HIV disease. A client with a CD4+ T-cell count between 200 and 499 cells/mm 3 is in the second stage of HIV disease. A client with a CD4+ T-cell count of less than 200 cells/mm 3 is in the third stage of HIV disease. The fourth stage of HIV disease indicates a confirmed HIV infection with no information regarding the CD4+ T-cell counts.

Which statement is true regarding Sjögren’s syndrome?
1
Sjögren’s syndrome increases lacrimation.
2
Sjögren’s syndrome increases body secretions.
3
Sjögren’s syndrome decreases the risk for infection.
Correct4
Sjögren’s syndrome decreases the digestion of carbohydrates.
Sjögren’s syndrome decreases the digestion of carbohydrates because of insufficient secretion of saliva. Sjögren’s syndrome decreases lacrimation. Sjögren’s syndrome also decreases body secretions and saliva, therefore increasing the risk of infection.

Confidence: Nailed It
Stats
2.
A client who underwent chemotherapy has leukopenia. Which instruction from the nurse will be beneficial for the client?
1
"You should avoid exposure to the sun."
2
"You should eat high-fiber foods and increase fluid intake."
Correct3
"You should avoid large crowds and people with infections."
4
"You should consume iron supplements and erythropoietin."
Low levels of white blood cells are called leukopenia. A leukopenic client should avoid large crowds and people with infection as the client may contract infection due to compromised immunity. The suggestion of avoiding exposure to the sun would be beneficial for a client with chemotherapy-induced skin changes. The suggestion of eating high-fiber foods and increasing fluid intake would be beneficial for a client with constipation after chemotherapy. Consuming iron supplements and erythropoietin would be required for a client who developed anemia after chemotherapy.

Confidence: Just a Guess
Stats
3.
After multiple bee stings, a client experiences an anaphylactic reaction. The nurse determines that the symptoms the client is experiencing are caused by what processes?
1
Respiratory depression and cardiac arrest
Correct2
Bronchial constriction and decreased peripheral resistance
3
Decreased cardiac output and dilation of major blood vessels
4
Constriction of capillaries and decreased peripheral circulation
Hypersensitivity to a foreign substance can cause an anaphylactic reaction; histamine is released, causing bronchial constriction, increased capillary permeability, and dilation of arterioles. This decreased peripheral resistance is associated with hypotension and inadequate circulation to major organs. Respiratory depression and cardiac arrest are the problems that result from bronchial constriction and vascular collapse. Dilation of arterioles occurs. Arterioles dilate, capillary permeability increases, and eventually vascular collapse occurs.

Which bacteria causes toxic shock syndrome in female clients?
1
Treponema pallidum
2
Streptococcus faecalis
Correct3
Staphylococcus aureus
4
Neisseria gonorrhoeae
Staphylococcus aureus causes toxic shock syndrome. Treponema pallidum causes syphilis. Streptococcus faecalis causes genitourinary tract infections and infection of surgical wounds. Neisseria gonorrhoeae causes gonorrhea and pelvic inflammatory disease.

A client making her first visit to the prenatal clinic asks which immunization can be administered safely to a pregnant woman. What should the nurse tell her?
1
Rubella (measles)
2
Rubeola (German measles)
Correct3
Inactive influenza
4
Varicella (chicken pox)
The inactive influenza and diphtheria, tetanus, pertussis (dTAP) immunizations can be safely administered during the first trimester of pregnancy, although dTAP is recommended at 27 to 36 weeks’ gestation to provide immunity to the mother and infant. The inactivated influenza vaccine may be given because it is a killed virus vaccine and will not have a teratogenic effect. Rubella (measles) and rubeola (German measles) vaccines are both live viruses that should never be administered during pregnancy because they can have teratogenic effects. Varicella (chicken pox) immunization is not given because it may cause birth defects in the fetus.

A primigravida has just given birth. The nurse is aware that the client has type AB Rh-negative blood. Her newborn’s blood type is B positive. What should the plan of care include?
1
Determining the father’s blood type
2
Preparing for a maternal blood transfusion
3
Observing the newborn for signs of ABO incompatibility
Correct4
Obtaining a prescription to administer Rho(D) immune globulin to the mother
Rho(D) immune globulin will prevent sensitization resulting from Rh incompatibility that may arise between an Rh-negative mother and an Rh-positive newborn. Determining the father’s blood type is unnecessary because only the mother’s and infant’s Rh factors are relevant. Preparing for a maternal blood transfusion is unnecessary; if a transfusion were needed, it would be for the newborn, not the mother. There is no ABO incompatibility; incompatibility might occur if the mother were O positive and the newborn had type A, B, or AB blood.

A nurse is counseling the family of a child with AIDS. What is the most important concern that the nurse should discuss with the parents?
1
Risk for injury
Correct2
Susceptibility to infection
3
Inadequate nutritional intake
4
Altered growth and development
Children with AIDS have a dysfunction of the immune system (depressed or ineffective T lymphocytes, B lymphocytes, and immunoglobulins) and are susceptible to opportunistic infections. All children are subject to injury because of their curiosity, inexperience, and lack of judgment. Although inadequate nutrition can be a problem for children with AIDS, the prevention of infection is the priority. Although children with AIDS are usually small for age, altered growth and development is not as life threatening as an infection.

Which dietary modifications can help improve the nutritional status of a client with acquired immunodeficiency syndrome (AIDS)?
Correct1
Refraining from consuming fatty foods
2
Refraining from consuming frequent meals
3
Refraining from consuming high-calorie foods
4
Refraining from consuming high-protein foods
Many clients with AIDS become intolerant to fat due to the disease and the antiretroviral medications. Therefore the client should be instructed to refrain from consuming fatty foods. The client should be encouraged to eat small and frequent meals to improve nutritional status. High-calorie and high-protein foods are beneficial to clients with AIDS because they provide energy and build immunity.

Which virus is responsible for causing infectious mononucleosis in clients?
1
Parvovirus
2
Coronavirus
3
Rotavirus
Correct4
Epstein-Barr virus
Epstein-Barr virus is responsible for mononucleosis and possibly Burkitt’s lymphoma. Parvovirus and rotavirus cause gastroenteritis. Corona virus causes upper respiratory tract infections.

Which is the first medication approved to reduce the risk of human immunodeficiency virus (HIV) infection in unaffected individuals?
Correct1
Truvada
2
Abacavir
3
Cromolyn
4
Methdilazine
Truvada is the first medication approved to reduce the risk of HIV infection in unaffected individuals who are at a high risk of HIV infection. Abacavir is administered to treat HIV infection and is a reverse transcriptase inhibitor. Cromolyn is administered in the management of allergic rhinitis and asthma. Methdilazine, an antihistamine, is administered to treat the skin and provide relief from itching.

Which medication should be immediately started when a client has an anaphylactic attack?
Correct1
Isoproterenol
Incorrect2
Diphenhydramine HCl
3
Hydrocortisone sodium succinate
4
Methylprednisolone sodium succinate
Isoproterenol is a beta-adrenergic, sympathomimetic drug that is considered to be a first-line of medication for the management of anaphylaxis. Diphenhydramine HCl is a second-line antihistamine to be used after the client is stabilized. Hydrocortisone sodium succinate and methylprednisolone sodium succinate are second-line corticosteroid medications that inhibit inflammatory mediators.

What is the function of IgG in the body?
1
Activates the degranulation of mast cells
Correct2
Activates the classic complement pathway
3
Prevents upper respiratory tract infections
Incorrect4
Prevents lower respiratory tract infections
The classic complement pathway is activated by the IgG and IgM antibodies. IgE antibodies cause a degranulation of mast cells. IgA antibodies are found largely in mucous membrane secretions and play an important role in preventing upper and lower respiratory tract infections.

Which medication should be immediately started when a client has an anaphylactic attack?
Correct1
Isoproterenol
Incorrect2
Diphenhydramine HCl
3
Hydrocortisone sodium succinate
4
Methylprednisolone sodium succinate
Isoproterenol is a beta-adrenergic, sympathomimetic drug that is considered to be a first-line of medication for the management of anaphylaxis. Diphenhydramine HCl is a second-line antihistamine to be used after the client is stabilized. Hydrocortisone sodium succinate and methylprednisolone sodium succinate are second-line corticosteroid medications that inhibit inflammatory mediators.

Confidence: Nailed It
Stats
12.
What is the function of IgG in the body?
1
Activates the degranulation of mast cells
Correct2
Activates the classic complement pathway
3
Prevents upper respiratory tract infections
Incorrect4
Prevents lower respiratory tract infections

What are the mediators of injury in IgE-mediated hypersensitivity reactions? Select all that apply.
1
Cytokines
Correct2
Mast cells
Correct3
Histamines
4
Neutrophils
Correct5
Leukotrienes
Mast cells, histamines, and leukotrienes are the mediators of injury in IgE-mediated hypersensitivity reactions. Cytokines are the mediators of injury in the delayed type of hypersensitivity reaction. Neutrophils are the mediators of injury in the immune-complex type of hypersensitivity reaction.

Which cytokine medication is administered to treat chemotherapy-induced neutropenia?
Correct1
Filgrastim
2
Oprelvekin
3
Aldesleukin
4
Darbepoetin alfa
Colony-stimulating factors such as filgrastim are administered to treat chemotherapy-induced neutropenia. Oprelvekin is used to prevent thrombocytopenia. Aldesleukin is used to treat metastatic renal cell carcinoma. Darbepoetin alfa is administered to treat anemia related to chronic cancer and anemia related to chronic kidney disease.

The nurse is teaching parents about the side effects of immunization vaccines. What expected side effect associated with the Haemophilus influenzae (Hib) vaccine will the nurse include in the teaching?
1
Urticaria
2
Lethargy
Correct3
Low-grade fever
4
Generalized rash
The Hib vaccine may cause a low-grade fever as the body reacts to the vaccine. Urticaria is more likely to occur with the tetanus and pertussis vaccines. Lethargy is not expected. There may be a mild reaction at the injection site, but a generalized rash is not expected.

A client is diagnosed with psoriasis, and the nurse is providing health teaching concerning skin care at home. Which recommendation does the nurse include in the teaching?
1
"Shower twice a day."
2
"Soak the affected areas in hot water."
Correct3
"Apply moisturizing lotion several times a day."
4
"Cover affected areas when in contact with others."
Moisturizing lotions provide an occlusive film on the skin surface so that usual water loss through the skin is limited, allowing the trapped water to hydrate the stratum corneum. Excessive exposure to water, particularly hot water, increases irritation and scaling. Psoriasis is not a communicable disease, and affected areas do not need to be covered when in contact with others.

Which medications are administered to inhibit purine synthesis and suppress cell-mediated and humoral immune responses? Select all that apply.
1
Sirolimus
Correct2
Azathioprine
3
Cyclophosphamide
4
Methylprednisolone
Correct5
Mycophenolate mofetil
Azathioprine and mycophenolate mofetil are administered to inhibit purine synthesis and suppress cell-mediated and humoral immunity. Sirolimus binds to a mammalian target of rapamycin (mTOR), which suppresses T-cell activation and proliferation. Cyclophosphamide is administered to treat cancers, autoimmune disorders, and amyloidosis. Methylprednisolone is a corticosteroid that inhibits cytokine production.

Which type of vaccines triggers the recipient’s immune system to produce antitoxins?
Incorrect1
Live vaccines
2
Killed vaccines
Correct3
Toxoid vaccines
4
Specific immune globulins
Toxoids are a nontoxic form of bacterial toxins that induce the formation of antitoxins by the recipient’s immune system. Examples of these types of vaccines include tetanus toxoid and diphtheria toxoid. Both live and killed vaccines induce the body’s immune system to produce antibodies, making the individual actively immune to the microbes. Specific immune globulins are high concentration of antibodies prepared from donated blood directed against specific antigens.

Which complication will the nurse suspect in a client with genital herpes disease?
1
Infertility
Correct2
Cold sores
Incorrect3
Reactive arthritis
4
Bartholin’s abscess
Cold sores are the autoinoculation of the virus to extragenital sites, such as the fingers and lips. It is a complication of genital herpes disease. Infertility and reactive arthritis are the complications of chlamydial infection. Bartholin’s abscess is a complication of gonorrhea.

Which parameter should the nurse consider while assessing the psychologic status of a client with acquired immune deficiency syndrome (AIDS)?
1
Sleep pattern
2
Severity of pain
Incorrect3
Cognitive changes
Correct4
Presence of anxiety
Presence of anxiety should be considered while assessing the psychologic status of a client with AIDS. Sleep patterns and severity of pain are related to the assessment of activity and rest, a physical status. Cognitive changes are related to the assessment of neurologic status.

Which malnutrition condition may predispose a client to secondary immunodeficiency?
Correct1
Cachexia
2
Cirrhosis
3
Diabetes mellitus
Incorrect4
Hodgkin’s lymphoma
Cachexia is a nutrition disorder that may occur due to wasting of muscle mass and weight, resulting in secondary immunodeficiency disorder. Cirrhosis, diabetes mellitus, and Hodgkin’s lymphoma also lead to secondary immunodeficiency disorder, but these are not malnutrition disorders.

What is the role of shark cartilage in the management of human immunodeficiency (HIV) and acquired immunodeficiency syndrome (AIDS)?
Incorrect1
Shark cartilage enhances immunity
2
Shark cartilage reduces oral thrush
Correct3
Shark cartilage is a complementary therapy
4
Shark cartilage is a nutritional supplement
Shark cartilage is considered as an alternative or complementary therapy to prescribed medications for clients with HIV and AIDS. Lymphocyte transfusions and bone marrow transplants are used to improve immunity in clients with HIV and AIDS. Lemon juice and lemongrass may provide relief from oral thrush in some clients with HIV and AIDS. A high-calorie, high-protein diet is advised to clients with HIV and AIDS to improve their nutritional status.

Confidence: Just a Guess
Stats
10.
The nurse is educating a client about protease inhibitors. What statement about protease inhibitors is true?
1
Protease inhibitors prevent viral replication.
Incorrect2
Protease inhibitors prevent the interaction between viral material and the CD4+ T-cell.
3
Protease inhibitors prevent viral and host genetic material integration.
Correct4
Protease inhibitors prevent the clipping of the viral strands into small functional pieces.
Protease inhibitors act by preventing the newly formed viral strands within the host CD4+ T-cell from being clipped into smaller functional pieces. Nucleoside reverse transcriptase inhibitors (NRTIs) prevent viral replication. NRTIs inhibits the transformation of viral single-stranded ribonucleic acid into host double-stranded deoxyribonucleic acid (DNA) by the action of the enzyme reverse transcriptase. Entry inhibitor drugs prevent the binding of the virus to the CD4 receptors. Integrase inhibitor drugs prevent the integration of viral material into the host’s DNA by the action of the enzyme integrase.

A nurse is reviewing several charts. Which condition is an autoimmune disorder?
Incorrect1
Addison’s disease
2
Cushing’s syndrome
Correct3
Hashimoto’s disease
4
Sheehan’s syndrome
Hashimoto’s disease is an autoimmune disorder, wherein the immune system attacks the thyroid gland. Addison’s disease is caused by adrenal insufficiency. Cushing’s syndrome is caused by increased body levels of cortisol. Sheehan’s syndrome is hemorrhage-associated hypopituitarism after delivery of a child.

Which activity by the community nurse can be considered an illness prevention strategy?
1
Encouraging the client to exercise daily
Correct2
Arranging an immunization program for chicken pox
3
Teaching the community about stress management
4
Teaching the client about maintaining a nutritious diet
An illness prevention program protects people from actual or potential threats to health. A chickenpox immunization program is an illness prevention program. It motivates the community to prevent a decline in health or functional levels. A health promotion program encourages the client to maintain the present levels of health. The nurse promotes the health of the client by encouraging the client to exercise daily. Wellness education teaches people how to care for themselves in a healthy manner. The nurse provides wellness education by teaching about stress management. The nurse promotes the health of the client by teaching the client to maintain a nutritious diet.

Which organ-specific autoimmune disorder is associated with a client’s kidney?
1
Graves’ disease
2
Addison’s disease
Correct3
Goodpasture syndrome
4
Guillain-Barré syndrome
Goodpasture syndrome is an autoimmune disorder associated with the client’s kidney. Graves’ disease and Addison’s disease are autoimmune disorders associated with the endocrine system. Guillain-Barré syndrome is an autoimmune disorder associated with the central nervous system.

The parents of an infant ask a nurse why their baby is not receiving the measles, mumps, and rubella (MMR) vaccine at the same time that other immunizations are being given. What explanation should the nurse give about why the MMR vaccine is administered at 12 to 15 months of age?
1
There is an increased risk of side effects in infants.
Correct2
Maternal antibodies provide immunity for about 1 year.
3
It interferes with the effectiveness of vaccines given during infancy.
4
There are rare instances of these infections occurring during the first year of life.
Maternal antibodies to measles, mumps, and rubella infection persist in the infant until approximately 15 months of age. Side effects are no more common among infants than in toddlers. The measles vaccination does not interfere with the effectiveness of other vaccines. Although the measles, mumps, and rubella do occasionally occur after the administration of the MMR vaccine during the first year of life, the vaccine is not given during this time because of the presence of maternal antibodies.

What is the function of IgG in the body?
1
Activates the degranulation of mast cells
Correct2
Activates the classic complement pathway
3
Prevents upper respiratory tract infections
4
Prevents lower respiratory tract infections
The classic complement pathway is activated by the IgG and IgM antibodies. IgE antibodies cause a degranulation of mast cells. IgA antibodies are found largely in mucous membrane secretions and play an important role in preventing upper and lower respiratory tract infections.

A nurse is reviewing several charts. Which condition is an autoimmune disorder?
1
Addison’s disease
2
Cushing’s syndrome
Correct3
Hashimoto’s disease
4
Sheehan’s syndrome
Hashimoto’s disease is an autoimmune disorder, wherein the immune system attacks the thyroid gland. Addison’s disease is caused by adrenal insufficiency. Cushing’s syndrome is caused by increased body levels of cortisol. Sheehan’s syndrome is hemorrhage-associated hypopituitarism after delivery of a child.

Which process does the IgD immunoglobulin support?
1
Manifestation of allergic reactions
Incorrect2
Protection of the body’s mucous surfaces
Correct3
Differentiation of the B-lymphocytes
4
Provision of the primary immune response
IgD is present on the lymphocyte surface; this immunoglobulin differentiates B-lymphocytes. IgE causes symptoms of allergic reactions by adhering to mast cells and basophils. IgE also helps to defend the body against parasitic infections. IgA lines the mucous membranes and protects the body surfaces. IgM provides the primary immune response.

The nurse is counseling a client infected with human immunodeficiency virus (HIV) regarding prevention of HIV transmission. Which statement by the client indicates the nurse needs to follow up?
1
"I should abstain from sexual activity."
Correct2
"I can safely have anal sex without any barriers."
3
"I should get HIV counseling if planning for pregnancy."
4
"I will use condoms while having sexual intercourse."
The client with HIV should use barrier protection when engaging in insertive sexual activity such as anal, oral, and vaginal. Therefore the nurse should follow up to provide the client with the correct information. All the other statements are correct and need no follow up. Abstaining from all sexual activity is a safe way to eliminate the risk of exposure to HIV in semen and vaginal secretions. The client should undergo HIV counseling and routinely offer access to voluntary HIV-antibody testing when planning for pregnancy. The most commonly used barrier is a condom, which allows for protected intercourse.

Confidence: Nailed It
Stats
4.
Which medications are useful to treat psoriasis? Select all that apply.
Correct1
Psoralen
Correct2
Anthralin
3
Isotretinoin
4
Clindamycin
Correct5
Calcipotriene
Psoriasis is a chronic autoimmune dermatitis treated with a systemic photosensitizer such as psoralen and topical agents such as calcipotriene and anthralin. Isotretinoin and clindamycin are used to treat acne vulgaris.

The nurse administers an initial dose of Haemophilus influenzae type b (Hib) vaccine to a 2-month-old infant. When should the nurse administer the final dose of the vaccine to the infant?
1
6-8 months of age
2
8-10 months of age
Correct3
12-15 months of age
4
16-18 months of age
The Haemophilus influenzae type b (Hib) vaccine is administered in four doses, finishing at the age of 12-15 months. Following the first dose at 2 months, the second is administered at 4 months, and the third at 6 months. For the final dose, 6-8 months and 8-10 months would be too soon; 16-18 months would be too late.

What type of hypersensitivity reaction is the cause of systemic lupus erythematosus?
1
Type I
2
Type II
Correct3
Type III
4
Type IV
Systemic lupus erythematosus is an example of an immune complex-mediated, or type III, hypersensitive reaction. Anaphylaxis is an example of a type I or immediate hypersensitive reaction. Cytotoxic or type II hypersensitive reactions can result in conditions such as myasthenia gravis and Goodpasture syndrome. Graft rejection and sarcoidosis are conditions that are caused by delayed or type IV hypersensitivity reactions.

Which are examples of a type IV hypersensitivity reaction? Select all that apply.
Correct1
Poison ivy allergic reaction
Correct2
Sarcoidosis
3
Myasthenia gravis
4
Rheumatoid arthritis
Incorrect5
Systemic lupus erythematosus
Sarcoidosis and poison ivy reactions are examples of type IV hypersensitivity reactions. In type IV hypersensitivity, the inflammation is caused by a reaction of sensitized T cells with the antigen and the resultant activation of macrophages due to lymphokine release. Myasthenia gravis is an example of a type II or cytotoxic hypersensitivity reaction. Rheumatoid arthritis and systemic lupus erythematosus are examples of type III immune complex-mediated reactions.

Confidence: Just a Guess
Stats
12.
Which virus can cause encephalitis in adults and children?
1
Rubella virus
Incorrect2
Parvovirus
3
Rotaviruses
Correct4
West Nile virus
The West Nile virus causes encephalitis. German measles is caused by rubella. Gastroenteritis is caused by parvovirus. Rotavirus also causes gastroenteritis.

A nurse is caring for a 13-year-old child who has an external fixation device on the leg. What is the nurse’s priority goal when providing pin care?
1
Easing pain
2
Minimizing scarring
Correct3
Preventing infection
4
Preventing skin breakdown
Pin sites provide a direct avenue for organisms into the bone. Pin care will not ease pain. Some scarring will occur at the pin insertion site regardless of pin site care. Skin has a tendency to grow around the pin, rather than break down, as long as infection is prevented.

When is the first dose of Rotarix vaccine administered in infants?
1
Birth to 6 weeks
Correct2
6-12 weeks
3
12-18 weeks
4
18-24 weeks
The Rotarix vaccine prevents rotavirus gastroenteritis and diarrhea-related problems. The vaccination requires two doses. The first dose of Rotarix should be given between 6 and 12 weeks followed by the second dose, which is given four or more weeks after the first dose. Administering the vaccine before the age of 6 weeks is too early, and administering it at 12-18 weeks or 18-24 weeks is too late.

Which parameter should the nurse consider while assessing the psychologic status of a client with acquired immune deficiency syndrome (AIDS)?
1
Sleep pattern
2
Severity of pain
3
Cognitive changes
Correct4
Presence of anxiety
Presence of anxiety should be considered while assessing the psychologic status of a client with AIDS. Sleep patterns and severity of pain are related to the assessment of activity and rest, a physical status. Cognitive changes are related to the assessment of neurologic status.

What is the function of a client’s natural killer cells?
1
Secrete immunoglobulins in response to the presence of a specific antigen
2
Heighten selectively and destroy non-self cells, including virally infected cells
3
Enhance immune activity through secretion of various factors, cytokines, and lymphokines
Correct4
Attack non-selectively on non-self cells, especially mutated and malignant cells
Natural killer cells attack non-selectively on non-self cells, especially body cells that have undergone mutation and become malignant. Plasma cells secrete immunoglobulins in response to the presence of a specific antigen. Cytotoxic T-cells attack selectively and destroy non-self cells, including virally infected cells. Helper T-cells enhance immune activity through secretion of various factors, cytokines, and lymphokines.

Which autoantigens are responsible for the development of Crohn’s disease?
Correct1
Crypt epithelial cells
2
Thyroid cell surface
3
Basement membranes of the lungs
4
Basement membranes of the glomeruli
Crypt epithelial cells are considered to be the autoantigens responsible for Crohn’s disease. Thyroid cell surfaces are autoantigens responsible for Hashimoto’s thyroiditis. The pulmonary and glomerular basement membranes act as autoantigens responsible for Goodpasture syndrome.

A nurse is caring for a client with pruritic lesions from an IgE-mediated hypersensitivity reaction. Which mediator of injury is involved?
Correct1
Histamine
2
Cytokine
3
Neutrophil
4
Macrophage
Histamine is one of the mediators of injury involving IgE-mediated injury that may cause pruritus. Cytokines are the mediators of injury in delayed hypersensitivity reaction. Neutrophils are involved in immune complex-mediated hypersensitivity reactions. Macrophages in tissues are involved in cytotoxic reactions.

Confidence: Just a Guess
Stats
7.
A client who has acquired human immunodeficiency syndrome (HIV) develops bacterial pneumonia. On admission to the emergency department, the client’s PaO 2 is 80 mm Hg. When the arterial blood gases are drawn again, the level is determined to be 65 mm Hg. What should the nurse do first?
1
Prepare to intubate the client.
Correct2
Increase the oxygen flow rate per facility protocol.
3
Decrease the tension of oxygen in the plasma.
4
Have the arterial blood gases redone to verify accuracy.
This decrease in PaO 2 indicates respiratory failure; it warrants immediate medical evaluation. Most facilities have a protocol to increase the oxygen flow rate to keep oxygen saturation greater than 92%. The client PaO 2 of 65 mm Hg is not severe enough to intubate the client without first increasing flow rate to determine if the client improves. Decreasing the tension of oxygen in the plasma is inappropriate and will compound the problem. The PaO 2 is a measure of the pressure (tension) of oxygen in the plasma; this level is decreased in individuals who have perfusion difficulties, such as those with pneumonia. Having the arterial blood gases redone to verify accuracy is negligent and dangerous; a falling PaO 2 level is a serious indication of worsening pulmonary status and must be addressed immediately. Drawing another blood sample and waiting for results will take too long.

A nurse is teaching parents of toddlers about why children receiving specific medications should not receive varicella vaccines. Which medication will be included in the discussion?
1
Insulin
Correct2
Steroids
3
Antibiotics
4
Anticonvulsants
Steroids have an immunosuppressive effect. It is thought that resistance to certain viral diseases, including varicella, is greatly decreased when a child takes steroids regularly. There is no known correlation between varicella and insulin. Because varicella is a viral disease, antibiotics will have no effect. There is no known correlation between varicella and anticonvulsants.

Confidence: Just a Guess
Stats
11.
A client is admitted to the hospital with a diagnosis of acute Guillain-Barré syndrome. Which assessment is priority?
1
Urinary output
2
Sensation to touch
3
Neurologic status
Correct4
Respiratory exchange
The respiratory center in the medulla oblongata can be affected with acute Guillain-Barré syndrome because the ascending paralysis can reach the diaphragm, leading to death from respiratory failure. Although urinary output, sensation to touch, and neurologic status are important, none of them are the priority.

Confidence: Pretty Sure
Stats
12.
A nurse is caring for a client with pulmonary tuberculosis. What must the nurse determine before discontinuing airborne precautions?
1
Client no longer is infected.
2
Tuberculin skin test is negative.
Correct3
Sputum is free of acid-fast bacteria.
4
Client’s temperature has returned to normal.
The absence of bacteria in the sputum indicates that the disease can no longer be spread by the airborne route. Treatment is over an extended period; eventually the client may not have an active disease, but still remains infected. Once an individual has been infected, the test will always be positive. The client’s temperature returning to normal is not evidence that the disease cannot be transmitted.

A laboratory report shows that a client tested positive for human epidermal growth factor (HER), and a medical report reveals the presence of advanced breast cancer. Which medication would be used to treat this condition?
1
Erlotinib
Correct2
Lapatinib
3
Rituximab
4
Tositumomab
HER-2 is overexpressed in clients with advanced breast cancer. Lapatinib inhibits epidermal growth factor-r (EGFR)-tyrosine kinase (TK) and binds HER-2. Erlotinib is an EFGR-TK inhibitor prescribed to treat non-small cell lung cancer and advanced pancreatic cancer. Rituximab and tositumomab are administered to treat non-Hodgkin’s lymphoma.

A client is admitted to the hospital with severe diarrhea, abdominal cramps, and vomiting after eating. These symptoms have lasted 5 days. Upon further assessment, the primary healthcare provider finds that the symptoms occurred after the client ate eggs, salad dressings, and sandwich fillings. Which food borne disease would be suspected in this client?
1
Listeriosis
2
Shigellosis
Correct3
Salmonellosis
4
Staphylococcus
A client with salmonellosis will experience severe diarrhea, abdominal cramps, and vomiting; these symptoms last as long as 5 days after the intake of contaminated food. This disorder may be caused by Salmonella typhi or Salmonella paratyphi. The causative organism is usually present in such foods as eggs, salad dressings, and sandwich fillings. A client with listeriosis will experience severe diarrhea, fever, headache, pneumonia, meningitis, and endocarditis 3 to 21 days after infection. The symptoms of shigellosis range from cramps and diarrhea to a fatal dysentery that lasts for 3 to 14 days. Pain, vomiting, diarrhea, perspiration, headache, fever, and prostration lasting for 1 or 2 days are the symptoms of a Staphylococcus infection.

A nurse is teaching a group of healthcare workers about the prevention of human immunodeficiency virus (HIV) transmission by healthcare workers. Which statements made by a healthcare worker indicate the need for the further teaching? Select all that apply.
Correct1
"I should never re-use equipment used in invasive procedures."
2
"If I perform exposure-prone procedures, I should know my HIV antibody status."
3
"I should identify exposure-prone procedures by institutions where they are performed."
Correct4
"If I am infected with HIV, I am restricted from practice of non-exposure-prone procedures."
5
"If I have exudative lesions or weeping dermatitis, I should not perform direct client care or handle client care equipment."
Workers must follow guidelines for disinfection and sterilization of reusable equipment used in invasive procedures. Workers infected with HIV are not restricted from practice of non-exposure-prone procedures, as long as they comply with standard precautions and sterilization and disinfection recommendations. Workers who are exposed to exposure-prone procedures should know their personal HIV antibody status. Workers should identify exposure-prone procedures by institutions where they are performed. Workers with exudative lesions or weeping dermatitis should not perform direct client care or handle client care equipment.

The 8-year-old son of migrant farm workers is brought to the county health clinic. He has no history of immunizations. After measles is diagnosed, the public health nurse goes to the migrant camp to search for people with a greater-than-average chance of contracting the disease. What is the name of this variable?
Correct1
Risk factor
2
Frequency rate
3
Probability rate
4
Causative factor
Risk factor is the term given to those factors that identify the target population so community resources may be used in the best interest of all. Frequency rate is an incorrect term for this variable; the nurse is not interested in how frequently measles occurs in migrant workers’ camps, which involves epidemiological research. Probability rate is the incorrect term for this variable; the correct term is risk factor, not probability rate. Causative factor is the term given to the cause of a given disease if it is known.

A client’s laboratory report shows severe neutropenia and thrombocytopenia. Which medication may have caused this condition?
1
Daclizumab
2
Cyclosporine
3
Methylprednisolone
Correct4
Mycophenolate mofetil
Mycophenolate mofetil is a cytotoxic drug that may cause neutropenia and thrombocytopenia. Daclizumab may cause hypersensitivity reaction and anaphylaxis. Cyclosporine may cause neurotoxicity, nephrotoxicity, and hypertension. Methylprednisolone may cause peptic ulcers, osteoporosis, and hyperglycemia.

Which symptoms are observed in a client with Sjögren’s syndrome? Select all that apply.
Incorrect1
Angioedema
Correct2
Tooth decay
Correct3
Corneal ulcers
Correct4
Vaginal dryness
5
Pulmonary hemorrhage
A client with Sjögren’s syndrome (SS) may have antigens specific to certain tissue types, such as HLA-DRW52, HLA-DR3, and HLA-B8. Sjögren’s syndrome may lead to autoimmune destruction of the lacrimal, salivary, and vaginal mucus-producing glands. Insufficient saliva decreases digestion of carbohydrates, which may promote tooth decay. Insufficient tears cause inflammation and ulceration of the cornea. Vaginal dryness increases the risk for infection and causes painful sexual intercourse. Angioedema occurs with a type I hypersensitivity reaction that may occur within seconds after exposure to the allergen. Clients with Goodpasture syndrome may have lung and kidney problems. Pulmonary hemorrhage is associated with this syndrome.

A nurse in the pediatric clinic plans to administer a booster immunization for polio to a child. Which vaccine should the nurse administer?
1
Hib
Correct2
IPV
Incorrect3
OPV
4
DTaP
The current polio vaccine is the inactivated polio vaccine (IPV; Salk vaccine) that is injectable. Hib is the Haemophilus influenzae type b vaccine. OPV is the oral polio vaccine (Sabin vaccine); it is no longer administered because it is related to vaccine-associated polio paralysis. However, it is used in the worldwide effort to eliminate the virus in countries where it is endemic. DTaP is the diphtheria, tetanus, and acellular pertussis vaccine.

A registered nurse is educating a client who has just undergone thoracentesis on the manifestations of pneumothorax. Which statements made by the client indicate effective learning? Select all that apply.
Correct1
"I’ll report any instance of blue skin right away."
Correct2
"I’ll report any feeling of air hunger immediately."
Incorrect3
"I’ll report any decrease in heart rate immediately."
4
"I’ll call you right away if my nagging cough disappears."
5
"I’ll call you right away if my shallow breathing goes away."
Client teaching regarding the manifestations of pneumothorax, partial or complete collapse of the lung, which may occur in the 24 hours after thoracentesis, is important. Manifestations that require immediate contact with the nearest emergency department are cyanosis and a feeling of air hunger. Other findings that must be reported include rapid heart rate (not decreased), a new-onset nagging cough (rather than the disappearance of such a cough), and rapid and shallow respirations (not the cessation of such respirations).

The nurse is caring for a client with human immunodeficiency virus (HIV) infection. Which clinical manifestations in the client should be immediately reported to the primary healthcare provider?
Correct1
Blood in the urine
2
New or productive cough
Incorrect3
Vomiting accompanied by fever
4
Burning, itching, and discharge from the eyes
A client with HIV infection is at risk for multiple diseases. Therefore blood in the urine should be reported immediately to the primary healthcare provider as it maybe life threatening. New or productive cough is not a life-threatening symptom and can be reported within 24 hours. Vomiting accompanied by fever is not a life-threatening symptom and can be reported within 24 hours. Burning, itching, and discharge from the eyes are not life threatening and can be reported within 24 hours.

A client has received ABO-incompatible blood from a donor by mistake. Which type of hypersensitivity reaction will occur in the client?
1
Type I
Correct2
Type II
Incorrect3
Type III
4
Type IV
A classic type II reaction occurs when a recipient receives ABO-incompatible blood from a donor. Naturally acquired antibodies to antigens of the ABO blood group are in the recipient’s serum but are not present on the erythrocyte membranes. Anaphylactic reactions are type I reactions that occur only in susceptible people who are highly sensitized to specific allergens. Tissue damage in immune-complex reactions, which are type III reactions, occur secondary to antigen-antibody complexes. Although cell-mediated responses are usually protective mechanisms, tissue damage occurs in delayed hypersensitivity reactions. The tissue damage in a type IV reaction does not occur in the presence of antibodies or complement.

Confidence: Pretty Sure
Stats
11.
Which conditions in clients are examples of cell-mediated immunity? Select all that apply.
Correct1
Tuberculosis
Correct2
Graft rejection
3
Allergic rhinitis
Correct4
Contact dermatitis
5
Anaphylactic shock
Conditions such as tuberculosis, graft rejection, contact dermatitis, and fungal infections are examples of cell-mediated immunity. T lymphocytes and macrophages sensitize T cells and cytokines to provide protection against fungus, viruses (intracellular), chronic infectious agents, and tumor cells. Allergic rhinitis and anaphylactic shock are examples of humoral immunity that are mediated by antibodies released by B lymphocytes.

The T lymphocytes in a healthy older adult client are 120 cells/mm 3 (120 cells/uL). The chest x-ray reveals shrinkage of the thymus gland. What might have led to the client’s condition?
Incorrect1
Immunodeficiency
Correct2
Immunosenescence
3
Immunosuppression
4
Immunocompetence
Immunosenescence is the primary cause of thymic involution (shrinking) and decrease in number of T cells to 120 cells/mm 3 (120 cells/uL) in an older client (normal range in healthy clients: 500 cells/mm 3 [500 cells/uL] to 1,200 cells/mm 3 [1,200 cells/uL]). Immunodeficiency is characterized by lack of or diminished reaction to an antigen or a group of antigens. Immunosuppression is a reduction of the activation or efficacy of the immune system, which occurs due to adverse reactions to treatment of other conditions. Immunocompetence exists when the body’s immune system can identify and inactivate or destroy foreign substances.

Share This
Flashcard

More flashcards like this

NCLEX 10000 Integumentary Disorders

When assessing a client with partial-thickness burns over 60% of the body, which finding should the nurse report immediately? a) ...

Read more

NCLEX 300-NEURO

A client with amyotrophic lateral sclerosis (ALS) tells the nurse, "Sometimes I feel so frustrated. I can’t do anything without ...

Read more

NASM Flashcards

Which of the following is the process of getting oxygen from the environment to the tissues of the body? Diffusion ...

Read more

Unfinished tasks keep piling up?

Let us complete them for you. Quickly and professionally.

Check Price

Successful message
sending